Programs & Examples On #Excel formula

This tag is for Microsoft Excel questions where the question or answers are specifically about the structure, syntax, or output of an Excel formula.

Excel Formula: Count cells where value is date

There is no interactive solution in Excel because some functions are not vector-friendly, like CELL, above quoted. For example, it's possible counting all the numbers whose absolute value is less than 3, because ABS is accepted inside a formula array.

So I've used the following array formula (Ctrl+Shift+Enter after edit with no curly brackets)

             ={SUM(IF(ABS(F1:F15)<3,1,0))}

If Column F has

                   F
          1 ...    2
          2 ....   4
          3 ....  -2
          4 ....   1
          5 .....  5

It counts 3! (-2,2 and 1). In order to show how ABS is array-friendly function let's do a simple test: Select G1:G5, digit =ABS(F1:F5) in the formula bar and press Ctrl+Shift+Enter. It's like someone write Abs(F1:F5)(1), Abs(F1:F5)(2), etc.

                   F    G
          1 ...    2  =ABS(F1:F5) => 2 
          2 ....   4  =ABS(F1:F5) => 4
          3 ....  -2  =ABS(F1:F5) => 2
          4 ....   1  =ABS(F1:F5) => 1
          5 .....  5  =ABS(F1:F5) => 5

Now I put some mixed data, including 2 date values.

                   F
          1 ...    Fab-25-2012
          2 ....   4
          3 ....   May-5-2013
          4 ....   Ball
          5 .....  5

In this case, CELL fails and return 1

             ={SUM(IF(CELL("format",F1:F15)="D4",1,0))}

It happens because CELL return the format of first cell of the range. (D4 is a m-d-y format)

So the only thing left is programming! A UDF(User defined Function) for formula array must return a variant array:

Function TypeCell(R As Range) As Variant
Dim V() As Variant
Dim Cel As Range
Dim I As Integer
Application.Volatile '// For revaluation in interactive environment
ReDim V(R.Cells.Count - 1) As Variant
I = 0
For Each Cel In R
   V(I) = VarType(Cel) '// Output array has the same size of input range.
   I = I + 1
Next Cel
TypeCell = V
End Function

Now is easy (the constant VbDate is 7):

             =SUM(IF(TypeCell(F1:F5)=7,1,0))  

It shows 2. That technique can be used for any shape of cells. I've tested vertical, horizontal and rectangular shapes, since you fill using for each order inside the function.

IF - ELSE IF - ELSE Structure in Excel

Say P7 is a Cell then you can use the following Syntex to check the value of the cell and assign appropriate value to another cell based on this following nested if:

=IF(P7=0,200,IF(P7=1,100,IF(P7=2,25,IF(P7=3,10,IF((P7=4),5,0)))))

Excel: VLOOKUP that returns true or false?

You still have to wrap it in an ISERROR, but you could use MATCH() instead of VLOOKUP():

Returns the relative position of an item in an array that matches a specified value in a specified order. Use MATCH instead of one of the LOOKUP functions when you need the position of an item in a range instead of the item itself.

Here's a complete example, assuming you're looking for the word "key" in a range of cells:

=IF(ISERROR(MATCH("key",A5:A16,FALSE)),"missing","found")

The FALSE is necessary to force an exact match, otherwise it will look for the closest value.

Check if an excel cell exists on another worksheet in a column - and return the contents of a different column

You can use following formulas.

For Excel 2007 or later:

=IFERROR(VLOOKUP(D3,List!A:C,3,FALSE),"No Match")

For Excel 2003:

=IF(ISERROR(MATCH(D3,List!A:A, 0)), "No Match", VLOOKUP(D3,List!A:C,3,FALSE))

Note, that

  • I'm using List!A:C in VLOOKUP and returns value from column ? 3
  • I'm using 4th argument for VLOOKUP equals to FALSE, in that case VLOOKUP will only find an exact match, and the values in the first column of List!A:C do not need to be sorted (opposite to case when you're using TRUE).

If two cells match, return value from third

I think what you want is something like:

=INDEX(B:B,MATCH(C2,A:A,0))  

I should mention that MATCH checks the position at which the value can be found within A:A (given the 0, or FALSE, parameter, it looks only for an exact match and given its nature, only the first instance found) then INDEX returns the value at that position within B:B.

Substring in excel

What about using Replace all? Just replace All on bracket to space. And comma to space. And I think you can achieve it.

How do I combine the first character of a cell with another cell in Excel?

Not sure why no one is using semicolons. This is how it works for me:

=CONCATENATE(LEFT(A1;1); B1)

Solutions with comma produce an error in Excel.

excel formula to subtract number of days from a date

Assuming the original date is in cell A1:

=DATE(YEAR(A1), MONTH(A1), DAY(A1)-180)

Excel: Can I create a Conditional Formula based on the Color of a Cell?

Unfortunately, there is not a direct way to do this with a single formula. However, there is a fairly simple workaround that exists.

On the Excel Ribbon, go to "Formulas" and click on "Name Manager". Select "New" and then enter "CellColor" as the "Name". Jump down to the "Refers to" part and enter the following:

=GET.CELL(63,OFFSET(INDIRECT("RC",FALSE),1,1))

Hit OK then close the "Name Manager" window.

Now, in cell A1 enter the following:

=IF(CellColor=3,"FQS",IF(CellColor=6,"SM",""))

This will return FQS for red and SM for yellow. For any other color the cell will remain blank.

***If the value in A1 doesn't update, hit 'F9' on your keyboard to force Excel to update the calculations at any point (or if the color in B2 ever changes).

Below is a reference for a list of cell fill colors (there are 56 available) if you ever want to expand things: http://www.smixe.com/excel-color-pallette.html

Cheers.

::Edit::

The formula used in Name Manager can be further simplified if it helps your understanding of how it works (the version that I included above is a lot more flexible and is easier to use in checking multiple cell references when copied around as it uses its own cell address as a reference point instead of specifically targeting cell B2).

Either way, if you'd like to simplify things, you can use this formula in Name Manager instead:

=GET.CELL(63,Sheet1!B2)

Extract the last substring from a cell

Simpler would be: =TRIM(RIGHT(SUBSTITUTE(TRIM(A2)," ",REPT(" ",99)),99))

You can use A2 in place of TRIM(A2) if you are sure that your data doesn't contain any unwanted spaces.

Based on concept explained by Rick Rothstein: http://www.excelfox.com/forum/showthread.php/333-Get-Field-from-Delimited-Text-String

Sorry for being necroposter!

Excel - find cell with same value in another worksheet and enter the value to the left of it

The easiest way is probably with VLOOKUP(). This will require the 2nd worksheet to have the employee number column sorted though. In newer versions of Excel, apparently sorting is no longer required.

For example, if you had a "Sheet2" with two columns - A = the employee number, B = the employee's name, and your current worksheet had employee numbers in column D and you want to fill in column E, in cell E2, you would have:

=VLOOKUP($D2, Sheet2!$A$2:$B$65535, 2, FALSE)

Then simply fill this formula down the rest of column D.

Explanation:

  • The first argument $D2 specifies the value to search for.
  • The second argument Sheet2!$A$2:$B$65535 specifies the range of cells to search in. Excel will search for the value in the first column of this range (in this case Sheet2!A2:A65535). Note I am assuming you have a header cell in row 1.
  • The third argument 2 specifies a 1-based index of the column to return from within the searched range. The value of 2 will return the second column in the range Sheet2!$A$2:$B$65535, namely the value of the B column.
  • The fourth argument FALSE says to only return exact matches.

Excel how to find values in 1 column exist in the range of values in another

This is what you need:

 =NOT(ISERROR(MATCH(<cell in col A>,<column B>, 0)))  ## pseudo code

For the first cell of A, this would be:

 =NOT(ISERROR(MATCH(A2,$B$2:$B$5, 0)))

Enter formula (and drag down) as follows:

enter image description here

You will get:

enter image description here

How to loop in excel without VBA or macros?

Add more columns when you have variable loops that repeat at different rates. I'm not sure explicitly what you're trying to do, but I think I've done something that could apply.

Creating a single loop in Excel is prettty simple. It actually does the work for you. Try this on a new workbook

  1. Enter "1" in A1
  2. Enter "=A1+1" in A2

A3 will automatically be "=A2+1" as you drag down. The first steps don't have to be that explicit. Excel will automatically recognize the pattern and count if you just put "2" in A2, but if we want B1-B5 to be "100" and B5-B10 to be "200" (counting up the same way) you can see why knowing how to do it explicitly matters. In this scenario, You just enter:

  1. "100" in B1, drag through to B5 and
  2. "=B1+100" in B6

B7 will automatically be "=B2+100" etc. as you drag down, so basically it increases every 5 rows infinitely. To make a loop of numbers 1-5 in column A:

  1. Enter "=A1" in cell A6. As you drag down, it will automatically be "=A2" in cell A7, etc. because of the way that Excel does things.

So, now we have column A repeating numbers 1-5 while column B is increasing by 100 every 5 cells.You could make column B repeat, for instance, the numbers 100-900 in using the same method as you did with column A as a way to produce, for instance, each possible combination with multiple variables. Drag down the columns and they'll do it infinitely. I'm not explicitly addressing the scenario given, but if you follow the steps and understand them, the concept should give you an answer to the problem that involves adding more columns and concactinating or using them as your variables.

Add a "sort" to a =QUERY statement in Google Spreadsheets

Sorting by C and D needs to be put into number form for the corresponding column, ie 3 and 4, respectively. Eg Order By 2 asc")

How do I reference a cell within excel named range?

You can use Excel's Index function:

=INDEX(Age, 5)

Excel select a value from a cell having row number calculated

You could use the INDIRECT function. This takes a string and converts it into a range

More info here

=INDIRECT("K"&A2)

But it's preferable to use INDEX as it is less volatile.

=INDEX(K:K,A2)

This returns a value or the reference to a value from within a table or range

More info here

Put either function into cell B2 and fill down.

Excel: Searching for multiple terms in a cell

Another way

=IF(SUMPRODUCT(--(NOT(ISERR(SEARCH({"Gingrich","Obama","Romney"},C1)))))>0,"1","")

Also, if you keep a list of values in, say A1 to A3, then you can use

=IF(SUMPRODUCT(--(NOT(ISERR(SEARCH($A$1:$A$3,C1)))))>0,"1","")

The wildcards are not necessary at all in the Search() function, since Search() returns the position of the found string.

How to remove only 0 (Zero) values from column in excel 2010

I selected columns that I want to delete 0 values then clicked DATA > FILTER. In column's header there is a filter icon appears. I clicked on that icon and selected only 0 values and clicked OK. Only 0 values becomes selected. Finally clear content OR use DELETE button. Problem Solved!

In Excel, sum all values in one column in each row where another column is a specific value

If column A contains the amounts to be reimbursed, and column B contains the "yes/no" indicating whether the reimbursement has been made, then either of the following will work, though the first option is recommended:

=SUMIF(B:B,"No",A:A)

or

=SUMIFS(A:A,B:B,"No")

Here is an example that will display the amounts paid and outstanding for a small set of sample data.

 A         B            C                   D
 Amount    Reimbursed?  Total Paid:         =SUMIF(B:B,"Yes",A:A)
 $100      Yes          Total Outstanding:  =SUMIF(B:B,"No",A:A)
 $200      No           
 $300      No
 $400      Yes
 $500      No

Result of Excel calculations

How to roundup a number to the closest ten?

Use ROUND but with num_digits = -1

=ROUND(A1,-1)

Also applies to ROUNDUP and ROUNDDOWN

From Excel help:

  • If num_digits is greater than 0 (zero), then number is rounded to the specified number of decimal places.
  • If num_digits is 0, then number is rounded to the nearest integer.
  • If num_digits is less than 0, then number is rounded to the left of the decimal point.

EDIT: To get the numbers to always round up use =ROUNDUP(A1,-1)

Referencing value in a closed Excel workbook using INDIRECT?

There is definitively no way to do this with standard formulas. However, a crazy sort of answer can be found here. It still avoids VBA, and it will allow you to get your result dynamically.

  1. First, make the formula that will generate your formula, but don't add the = at the beginning!

  2. Let us pretend that you have created this formula in cell B2 of Sheet1, and you would like the formula to be evaluated in column c.

  3. Now, go to the Formulas tab, and choose "Define Name". Give it the name myResult (or whatever you choose), and under Refers To, write =evaluate(Sheet1!$B2) (note the $)

  4. Finally, go to C2, and write =myResult. Drag down, and... voila!

Simulate string split function in Excel formula

These things tend to be simpler if you write them a cell at a time, breaking the lengthy formulas up into smaller ones, where you can check them along the way. You can then hide the intermediate calculations, or roll them all up into a single formula.

For instance, taking James' formula:

=IFERROR(LEFT(A3, FIND(" ", A3, 1)), A3)

Which is only valid in Excel 2007 or later.

Break it up as follows:

B3: =FIND(" ", A3)
C3: =IF(ISERROR(B3),A3,LEFT(A3,B3-1))

It's just a little easier to work on, a chunk at a time. Once it's done, you can turn it into

=IF(ISERROR(FIND(" ", A3)),A3,LEFT(A3,FIND(" ", A3)-1))

if you so desire.

Excel compare two columns and highlight duplicates

The easiest way to do it, at least for me, is:

Conditional format-> Add new rule->Set your own formula:

=ISNA(MATCH(A2;$B:$B;0))

Where A2 is the first element in column A to be compared and B is the column where A's element will be searched.

Once you have set the formula and picked the format, apply this rule to all elements in the column.

Hope this helps

Simple Pivot Table to Count Unique Values

If you have the data sorted.. i suggest using the following formula

=IF(OR(A2<>A3,B2<>B3),1,0)

This is faster as it uses less cells to calculate.

Return values from the row above to the current row

You can also use =OFFSET([@column];-1;0) if you are in a named table.

Excel Formula which places date/time in cell when data is entered in another cell in the same row

Not sure if this works for cells with functions but I found this code elsewhere for single cell entries and modified it for my use. If done properly, you do not need to worry about entering a function in a cell or the file changing the dates to that day's date every time it is opened.

  • open Excel
  • press "Alt+F11"
  • Double-click on the worksheet that you want to apply the change to (listed on the left)
  • copy/paste the code below
  • adjust the Range(:) input to correspond to the column you will update
  • adjust the Offset(0,_) input to correspond to the column where you would like the date displayed (in the version below I am making updates to column D and I want the date displayed in column F, hence the input entry of "2" for 2 columns over from column D)
  • hit save
  • repeat steps above if there are other worksheets in your workbook that need the same code
  • you may have to change the number format of the column displaying the date to "General" and increase the column's width if it is displaying "####" after you make an updated entry

Copy/Paste Code below:


Private Sub Worksheet_Change(ByVal Target As Range)

If Intersect(Target, Range("D:D")) Is Nothing Then Exit Sub
Target.Offset(0, 2) = Date

End Sub


Good luck...

Showing percentages above bars on Excel column graph

You can do this with a pivot table and add a line with the pourcentage for each category like brettdj showed in his answer. But if you want to keep your data as it is, there is a solution by using some javascript.

Javascript is a powerful language offering a lot of useful data visualization libraries like plotly.js.

Here is a working code I have written for you:

https://www.funfun.io/1/#/edit/5a58c6368dfd67466879ed27

In this example, I use a Json file to get the data from the embedded spreadsheet, so I can use it in my javascript code and create a bar chart.

I calculate the percentage by adding the values of all the category present in the table and using this formula (you can see it in the script.js file): Percentage (%) = 100 x partial value / total value

It automatically calculates the total and pourcentage even if you add more categories.

I used plotly.js to create my chart, it has a good documentation and lots of examples for beginners, this code gets all the option you want to use:

var trace1 = {
    x: xValue, 
    y: data,
    type: 'bar',
    text: yValue,
    textposition: 'auto',
    hoverinfo: 'none',
    marker: {
    color: 'yellow',
    opacity: 0.6,
    line: {
      color: 'yellow',
      width: 1.5
    }
  }
};

It is rather self explanatory, the text is where you put the percentage.

Once you've made your chart you can load it in excel by passing the URL in the Funfun add-in. Here is how it looks like with my example:

final

I know it is an old post but I hope it helps people with the same problem !

Disclosure : I’m a developer of funfun

Remove leading or trailing spaces in an entire column of data

If it's the same number of characters at the beginning of the cell each time, you can use the text to columns command and select the fixed width option to chop the cell data into two columns. Then just delete the unwanted stuff in the first column.

How do I convert a calendar week into a date in Excel?

=(MOD(R[-1]C-1,100)*7+DATE(INT(R[-1]C/100+2000),1,1)-2)

yyww as the given week exp:week 51 year 2014 will be 1451

How to disable Excel's automatic cell reference change after copy/paste?

From http://spreadsheetpage.com/index.php/tip/making_an_exact_copy_of_a_range_of_formulas_take_2:

  1. Put Excel in formula view mode. The easiest way to do this is to press Ctrl+` (that character is a "backwards apostrophe," and is usually on the same key that has the ~ (tilde).
  2. Select the range to copy.
  3. Press Ctrl+C
  4. Start Windows Notepad
  5. Press Ctrl+V to past the copied data into Notepad
  6. In Notepad, press Ctrl+A followed by Ctrl+C to copy the text
  7. Activate Excel and activate the upper left cell where you want to paste the formulas. And, make sure that the sheet you are copying to is in formula view mode.
  8. Press Ctrl+V to paste.
  9. Press Ctrl+` to toggle out of formula view mode.

Note: If the paste operation back to Excel doesn't work correctly, chances are that you've used Excel's Text-to-Columns feature recently, and Excel is trying to be helpful by remembering how you last parsed your data. You need to fire up the Convert Text to Columns Wizard. Choose the Delimited option and click Next. Clear all of the Delimiter option checkmarks except Tab.

Or, from http://spreadsheetpage.com/index.php/tip/making_an_exact_copy_of_a_range_of_formulas/:

If you're a VBA programmer, you can simply execute the following code: 
With Sheets("Sheet1")
 .Range("A11:D20").Formula = .Range("A1:D10").Formula
End With

Format numbers in thousands (K) in Excel

The examples above use a 'K' an uppercase k used to represent kilo or 1000. According to wiki, kilo or 1000's should be represented in lower case. So, rather than £300K, use £300k or in a code example :-

[>=1000]£#,##0,"k";[red][<=-1000]-£#,##0,"k";0

In Excel how to get the left 5 characters of each cell in a specified column and put them into a new column

1) Put =Left(E1,5) in F1

2) Copy F1, then select entire F column and paste.

How to Convert Excel Numeric Cell Value into Words

There is no built-in formula in excel, you have to add a vb script and permanently save it with your MS. Excel's installation as Add-In.

  1. press Alt+F11
  2. MENU: (Tool Strip) Insert Module
  3. copy and paste the below code


Option Explicit

Public Numbers As Variant, Tens As Variant

Sub SetNums()
    Numbers = Array("", "One", "Two", "Three", "Four", "Five", "Six", "Seven", "Eight", "Nine", "Ten", "Eleven", "Twelve", "Thirteen", "Fourteen", "Fifteen", "Sixteen", "Seventeen", "Eighteen", "Nineteen")
    Tens = Array("", "", "Twenty", "Thirty", "Forty", "Fifty", "Sixty", "Seventy", "Eighty", "Ninety")
End Sub

Function WordNum(MyNumber As Double) As String
    Dim DecimalPosition As Integer, ValNo As Variant, StrNo As String
    Dim NumStr As String, n As Integer, Temp1 As String, Temp2 As String
    ' This macro was written by Chris Mead - www.MeadInKent.co.uk
    If Abs(MyNumber) > 999999999 Then
        WordNum = "Value too large"
        Exit Function
    End If
    SetNums
    ' String representation of amount (excl decimals)
    NumStr = Right("000000000" & Trim(Str(Int(Abs(MyNumber)))), 9)
    ValNo = Array(0, Val(Mid(NumStr, 1, 3)), Val(Mid(NumStr, 4, 3)), Val(Mid(NumStr, 7, 3)))
    For n = 3 To 1 Step -1    'analyse the absolute number as 3 sets of 3 digits
        StrNo = Format(ValNo(n), "000")
        If ValNo(n) > 0 Then
            Temp1 = GetTens(Val(Right(StrNo, 2)))
            If Left(StrNo, 1) <> "0" Then
                Temp2 = Numbers(Val(Left(StrNo, 1))) & " hundred"
                If Temp1 <> "" Then Temp2 = Temp2 & " and "
            Else
                Temp2 = ""
            End If
            If n = 3 Then
                If Temp2 = "" And ValNo(1) + ValNo(2) > 0 Then Temp2 = "and "
                WordNum = Trim(Temp2 & Temp1)
            End If
            If n = 2 Then WordNum = Trim(Temp2 & Temp1 & " thousand " & WordNum)
            If n = 1 Then WordNum = Trim(Temp2 & Temp1 & " million " & WordNum)
        End If
    Next n
    NumStr = Trim(Str(Abs(MyNumber)))
    ' Values after the decimal place
    DecimalPosition = InStr(NumStr, ".")
    Numbers(0) = "Zero"
    If DecimalPosition > 0 And DecimalPosition < Len(NumStr) Then
        Temp1 = " point"
        For n = DecimalPosition + 1 To Len(NumStr)
            Temp1 = Temp1 & " " & Numbers(Val(Mid(NumStr, n, 1)))
        Next n
        WordNum = WordNum & Temp1
    End If
    If Len(WordNum) = 0 Or Left(WordNum, 2) = " p" Then
        WordNum = "Zero" & WordNum
    End If
End Function

Function GetTens(TensNum As Integer) As String
' Converts a number from 0 to 99 into text.
    If TensNum <= 19 Then
        GetTens = Numbers(TensNum)
    Else
        Dim MyNo As String
        MyNo = Format(TensNum, "00")
        GetTens = Tens(Val(Left(MyNo, 1))) & " " & Numbers(Val(Right(MyNo, 1)))
    End If
End Function

After this, From File Menu select Save Book ,from next menu select "Excel 97-2003 Add-In (*.xla)

It will save as Excel Add-In. that will be available till the Ms.Office Installation to that machine.

Now Open any Excel File in any Cell type =WordNum(<your numeric value or cell reference>)

you will see a Words equivalent of the numeric value.

This Snippet of code is taken from: http://en.kioskea.net/forum/affich-267274-how-to-convert-number-into-text-in-excel

How do I recognize "#VALUE!" in Excel spreadsheets?

This will return TRUE for #VALUE! errors (ERROR.TYPE = 3) and FALSE for anything else.

=IF(ISERROR(A1),ERROR.TYPE(A1)=3)

Sum up a column from a specific row down

=Sum(C:C)-Sum(C1:C5)

Sum everything then remove the sum of the values in the cells you don't want, no Volatile Offset's, Indirect's, or Array's needed.

Just for fun if you don't like that method you could also use:

=SUM($C$6:INDEX($C:$C,MATCH(9.99999999999999E+307,$C:$C))

The above formula will Sum only from C6 through the last cell in C:C where a match of a number is found. This is also non-volatile, but I believe more costly and sloppy. Just added it in case you'd prefer this anyways.

If you would like to do function like CountA for text using the last text value in a column you could use.

=COUNTIF(C6:INDEX($C:$C,MATCH(REPT("Z",255),$C:$C)),"T")

you could also use other combinations like:

=Sum($C$6:$C$65536) 

or

=CountIF($C$6:$C$65536,"T") 

The above would do what you ask in Excel 2003 and lower

=Sum($C$6:$C$1048576) 

or

=CountIF($C$6:$C$1048576,"T")

Would both work for Excel 2007+

All above functions would simply ignore all the blank values under the last value.

How can I sort one set of data to match another set of data in Excel?

You could also simply link both cells, and have an =Cell formula in each column like, =Sheet2!A2 in Sheet 1 A2 and =Sheet2!B2 in Sheet 1 B2, and drag it down, and then sort those two columns the way you want.

  • If they don't sort the way you want, put the order you want to sort them in another column and sort all three columns by that.
  • If you drag it down further and get zeros you can edit the =Cell formula to show "" IF there is nothing. =(if(cell="","",cell)
  • Cutting, pasting, deleting, and inserting rows is something to be weary of. #REF! errors could occur.

This would be better if your unique items change also, then all you would do is sort and be done.

Convert date field into text in Excel

You can use TEXT like this as part of a concatenation

=TEXT(A1,"dd-mmm-yy") & " other string"

enter image description here

Excel formula to remove space between words in a cell

Suppose the data is in the B column, write in the C column the formula:

=SUBSTITUTE(B1," ","")

Copy&Paste the formula in the whole C column.

edit: using commas or semicolons as parameters separator depends on your regional settings (I have to use the semicolons). This is weird I think. Thanks to @tocallaghan and @pablete for pointing this out.

Excel: last character/string match in a string

How about creating a custom function and using that in your formula? VBA has a built-in function, InStrRev, that does exactly what you're looking for.

Put this in a new module:

Function RSearch(str As String, find As String)
    RSearch = InStrRev(str, find)
End Function

And your function will look like this (assuming the original string is in B1):

=LEFT(B1,RSearch(B1,"\"))

Error in finding last used cell in Excel with VBA

For the last 3+ years these are the functions that I am using for finding last row and last column per defined column(for row) and row(for column):

Last Column:

Function lastCol(Optional wsName As String, Optional rowToCheck As Long = 1) As Long

    Dim ws  As Worksheet

    If wsName = vbNullString Then
        Set ws = ActiveSheet
    Else
        Set ws = Worksheets(wsName)
    End If

    lastCol = ws.Cells(rowToCheck, ws.Columns.Count).End(xlToLeft).Column

End Function

Last Row:

Function lastRow(Optional wsName As String, Optional columnToCheck As Long = 1) As Long

    Dim ws As Worksheet

    If wsName = vbNullString Then
        Set ws = ActiveSheet
    Else
        Set ws = Worksheets(wsName)
    End If

    lastRow = ws.Cells(ws.Rows.Count, columnToCheck).End(xlUp).Row

End Function

For the case of the OP, this is the way to get the last row in column E:

Debug.Print lastRow(columnToCheck:=Range("E4:E48").Column)

Last Row, counting empty rows with data:

Here we may use the well-known Excel formulas, which give us the last row of a worksheet in Excel, without involving VBA - =IFERROR(LOOKUP(2,1/(NOT(ISBLANK(A:A))),ROW(A:A)),0)

In order to put this in VBA and not to write anything in Excel, using the parameters for the latter functions, something like this could be in mind:

Public Function LastRowWithHidden(Optional wsName As String, Optional columnToCheck As Long = 1) As Long

    Dim ws As Worksheet

    If wsName = vbNullString Then
        Set ws = ActiveSheet
    Else
        Set ws = Worksheets(wsName)
    End If

    Dim letters As String
    letters = ColLettersGenerator(columnToCheck)
    LastRowWithHidden = ws.Evaluate("=IFERROR(LOOKUP(2,1/(NOT(ISBLANK(" & letters & "))),ROW(" & letters & " )),0)")

End Function

Function ColLettersGenerator(col As Long) As String

    Dim result As Variant
    result = Split(Cells(1, col).Address(True, False), "$")
    ColLettersGenerator = result(0) & ":" & result(0)

End Function

Excel Date to String conversion

If you are not using programming then do the following (1) select the column (2) right click and select Format Cells (3) Select "Custom" (4) Just Under "Type:" type dd/mm/yyyy hh:mm:ss

Using the value in a cell as a cell reference in a formula?

Use INDIRECT()

=SUM(INDIRECT(<start cell here> & ":" & <end cell here>))

Check whether a cell contains a substring

Try using this:

=ISNUMBER(SEARCH("Some Text", A3))

This will return TRUE if cell A3 contains Some Text.

PivotTable to show values, not sum of values

Another easier way to do it is to upload your file to google sheets, then add a pivot, for the columns and rows select the same as you would with Excel, however, for values select Calculated Field and then in the formula type in =

In my case the column header is URL

Excel doesn't update value unless I hit Enter

Select all the data and use the option "Text to Columns", that will allow your data for Applying Number Formatting ERIK

Excel - programm cells to change colour based on another cell

Select ColumnB and as two CF formula rules apply:

Green: =AND(B1048576="X",B1="Y")

Red: =AND(B1048576="X",B1="W")

enter image description here

What's the quickest way to multiply multiple cells by another number?

Select Product from formula bar in your answer cell.

Select cells you want to multiply.

Change the fill color of a cell based on a selection from a Drop Down List in an adjacent cell

You can leverage Conditional Formatting as follows.

  1. In cell H8 select Format > Conditional Formatting...
  2. In Condition1, select Formula Is in first drop down menu
  3. In the next textbox type =I8="Elementary"
  4. Select Format... and select the color you want etc.
  5. Select Add>> and repeat steps 1 to 4

Note that you can only have (in excel 2003) three separate conditions so you will only be able to have different formatting for three items in the drop down menu. If the idea is to make them visually distinct then (maybe) having no color for one of the selections is not a problem?

If the cell is never blank, you can use format (not conditional) to get 4 distinct visuals.

Using "If cell contains #N/A" as a formula condition.

Input the following formula in C1:

=IF(ISNA(A1),B1,A1*B1)

Screenshots:

When #N/A:

enter image description here

When not #N/A:

enter image description here

Let us know if this helps.

How can I color dots in a xy scatterplot according to column value?

Recently I had to do something similar and I resolved it with the code below. Hope it helps!

Sub ColorCode()
Dim i As Integer
Dim j As Integer
i = 2
j = 1

Do While ActiveSheet.Cells(i, 1) <> ""


If Cells(i, 5).Value = "RED" Then
ActiveSheet.ChartObjects("YourChartName").Chart.FullSeriesCollection(1).Points(j).MarkerForegroundColor = RGB(255, 0, 0)



Else

If Cells(i, 5).Value = "GREEN" Then
ActiveSheet.ChartObjects("YourChartName").Chart.FullSeriesCollection(1).Points(j).MarkerForegroundColor = RGB(0, 255, 0)

Else

If Cells(i, 5).Value = "GREY" Then
ActiveSheet.ChartObjects("YourChartName").Chart.FullSeriesCollection(1).Points(j).MarkerForegroundColor = RGB(192, 192, 192)

Else

If Cells(i, 5).Value = "YELLOW" Then
ActiveSheet.ChartObjects("YourChartName").Chart.FullSeriesCollection(1).Points(j).MarkerForegroundColor = RGB(255, 255, 0)

End If
End If
End If
End If

i = i + 1
j = j + 1

Loop



End Sub

Excel formula to get week number in month (having Monday)

Jonathan from the ExcelCentral forums suggests:

=WEEKNUM(A1,2)-WEEKNUM(DATE(YEAR(A1),MONTH(A1),1),2)+1 

This formula extracts the week of the year [...] and then subtracts it from the week of the first day in the month to get the week of the month. You can change the day that weeks begin by changing the second argument of both WEEKNUM functions (set to 2 [for Monday] in the above example). For weeks beginning on Sunday, use:

=WEEKNUM(A1,1)-WEEKNUM(DATE(YEAR(A1),MONTH(A1),1),1)+1

For weeks beginning on Tuesday, use:

=WEEKNUM(A1,12)-WEEKNUM(DATE(YEAR(A1),MONTH(A1),1),12)+1

etc.

I like it better because it's using the built in week calculation functionality of Excel (WEEKNUM).

Converting time stamps in excel to dates

i got result from this in LibreOffice Calc :

=DATE(1970,1,1)+Column_id_here/60/60/24

Conditionally formatting if multiple cells are blank (no numerics throughout spreadsheet )

enter image description here

How about just > Format only cells that contain - in the drop down box select Blanks

Return empty cell from formula in Excel

This is how I did it for the dataset I was using. It seems convoluted and stupid, but it was the only alternative to learning how to use the VB solution mentioned above.

  1. I did a "copy" of all the data, and pasted the data as "values".
  2. Then I highlighted the pasted data and did a "replace" (Ctrl-H) the empty cells with some letter, I chose q since it wasn't anywhere on my data sheet.
  3. Finally, I did another "replace", and replaced q with nothing.

This three step process turned all of the "empty" cells into "blank" cells". I tried merging steps 2 & 3 by simply replacing the blank cell with a blank cell, but that didn't work--I had to replace the blank cell with some kind of actual text, then replace that text with a blank cell.

Excel formula to display ONLY month and year?

Very easy, trial and error. Go to the cell you want the month in. Type the Month, go to the next cell and type the year, something weird will come up but then go to your number section click on the little arrow in the right bottom and highlight text and it will change to the year you originally typed

How to compare two columns in Excel and if match, then copy the cell next to it

It might be easier with vlookup. Try this:

=IFERROR(VLOOKUP(D2,G:H,2,0),"")

The IFERROR() is for no matches, so that it throws "" in such cases.

VLOOKUP's first parameter is the value to 'look for' in the reference table, which is column G and H.

VLOOKUP will thus look for D2 in column G and return the value in the column index 2 (column G has column index 1, H will have column index 2), meaning that the value from column H will be returned.

The last parameter is 0 (or equivalently FALSE) to mean an exact match. That's what you need as opposed to approximate match.

Excel formula to reference 'CELL TO THE LEFT'

You could use a VBA script that changes the conditional formatting of a selection (you might have to adjust the condition & formatting accordingly):

For Each i In Selection
i.FormatConditions.Delete
i.FormatConditions.Add Type:=xlCellValue, Operator:=xlLess, Formula1:="=" & i.Offset(0, -1).Address
With i.FormatConditions(1).Font
    .Bold = True
End With
Next i

What does an exclamation mark before a cell reference mean?

If you use that forumla in the name manager you are creating a dynamic range which uses "this sheet" in place of a specific sheet.

As Jerry says, Sheet1!A1 refers to cell A1 on Sheet1. If you create a named range and omit the Sheet1 part you will reference cell A1 on the currently active sheet. (omitting the sheet reference and using it in a cell formula will error).

edit: my bad, I was using $A$1 which will lock it to the A1 cell as above, thanks pnuts :p

How to SUM parts of a column which have same text value in different column in the same row

If your data has the names grouped as shown then you can use this formula in D2 copied down to get a total against the last entry for each name

=IF((A2=A3)*(B2=B3),"",SUM(C$2:C2)-SUM(D$1:D1))

See screenshot

enter image description here

In Excel, how do I extract last four letters of a ten letter string?

No need to use a macro. Supposing your first string is in A1.

=RIGHT(A1, 4)

Drag this down and you will get your four last characters.

Edit: To be sure, if you ever have sequences like 'ABC DEF' and want the last four LETTERS and not CHARACTERS you might want to use trimspaces()

=RIGHT(TRIMSPACES(A1), 4)

Edit: As per brettdj's suggestion, you may want to check that your string is actually 4-character long or more:

=IF(TRIMSPACES(A1)>=4, RIGHT(TRIMSPACES(A1), 4), TRIMSPACES(A1))

Ignore Duplicates and Create New List of Unique Values in Excel

Find here mentioned above formula with error control

=IFERROR(INDEX($B$2:$B$9, MATCH(0,COUNTIF($D$1:D1, $B$2:$B$9), 0)),"") 

where: (B2:B9 is the column data which you want to extract the unique values, D1 is the above cell where your formula is located)

Count if two criteria match - EXCEL formula

Add the sheet name infront of the cell, e.g.:

=COUNTIFS(stock!A:A,"M",stock!C:C,"Yes")

Assumes the sheet name is "stock"

Conditionally formatting cells if their value equals any value of another column

Another simpler solution is to use this formula in the conditional formatting (apply to column A):

=COUNTIF(B:B,A1)

Regards!

Refer to a cell in another worksheet by referencing the current worksheet's name?

Here is how I made monthly page in similar manner as Fernando:

  1. I wrote manually on each page number of the month and named that place as ThisMonth. Note that you can do this only before you make copies of the sheet. After copying Excel doesn't allow you to use same name, but with sheet copy it does it still. This solution works also without naming.
  2. I added number of weeks in the month to location C12. Naming is fine also.
  3. I made five weeks on every page and on fifth week I made function

      =IF(C12=5,DATE(YEAR(B48),MONTH(B48),DAY(B48)+7),"")
    

    that empties fifth week if this month has only four weeks. C12 holds the number of weeks.

  4. ...
  5. I created annual Excel, so I had 12 sheets in it: one for each month. In this example name of the sheet is "Month". Note that this solutions works also with the ODS file standard except you need to change all spaces as "_" characters.
  6. I renamed first "Month" sheet as "Month (1)" so it follows the same naming principle. You could also name it as "Month1" if you wish, but "January" would require a bit more work.
  7. Insert following function on the first day field starting sheet #2:

     =INDIRECT(CONCATENATE("'Month (",ThisMonth-1,")'!B15"))+INDIRECT(CONCATENATE("'Month (",ThisMonth-1,")'!C12"))*7
    

    So in another word, if you fill four or five weeks on the previous sheet, this calculates date correctly and continues from correct date.

Excel: Search for a list of strings within a particular string using array formulas?

In case others may find this useful: I found that by adding an initial empty cell to my list of search terms, a zero value will be returned instead of error.

={INDEX(SearchTerms!$A$1:$A$38,MAX(IF(ISERROR(SEARCH(SearchTerms!$A$1:$A$38,SearchCell)),0,1)*((SearchTerms!$B$1:$B$38)+1)))}

NB: Column A has the search terms, B is the row number index.

Pass row number as variable in excel sheet

An alternative is to use OFFSET:

Assuming the column value is stored in B1, you can use the following

C1 = OFFSET(A1, 0, B1 - 1)

This works by:

a) taking a base cell (A1)
b) adding 0 to the row (keeping it as A)
c) adding (A5 - 1) to the column

You can also use another value instead of 0 if you want to change the row value too.

How to keep one variable constant with other one changing with row in excel

There are two kinds of cell reference, and it's really valuable to understand them well.

One is relative reference, which is what you get when you just type the cell: A5. This reference will be adjusted when you paste or fill the formula into other cells.

The other is absolute reference, and you get this by adding dollar signs to the cell reference: $A$5. This cell reference will not change when pasted or filled.

A cool but rarely used feature is that row and column within a single cell reference may be independent: $A5 and A$5. This comes in handy for producing things like multiplication tables from a single formula.

How to compare two columns in Excel (from different sheets) and copy values from a corresponding column if the first two columns match?

As kmcamara discovered, this is exactly the kind of problem that VLOOKUP is intended to solve, and using vlookup is arguably the simplest of the alternative ways to get the job done.

In addition to the three parameters for lookup_value, table_range to be searched, and the column_index for return values, VLOOKUP takes an optional fourth argument that the Excel documentation calls the "range_lookup".

Expanding on deathApril's explanation, if this argument is set to TRUE (or 1) or omitted, the table range must be sorted in ascending order of the values in the first column of the range for the function to return what would typically be understood to be the "correct" value. Under this default behavior, the function will return a value based upon an exact match, if one is found, or an approximate match if an exact match is not found.

If the match is approximate, the value that is returned by the function will be based on the next largest value that is less than the lookup_value. For example, if "12AT8003" were missing from the table in Sheet 1, the lookup formulas for that value in Sheet 2 would return '2', since "12AT8002" is the largest value in the lookup column of the table range that is less than "12AT8003". (VLOOKUP's default behavior makes perfect sense if, for example, the goal is to look up rates in a tax table.)

However, if the fourth argument is set to FALSE (or 0), VLOOKUP returns a looked-up value only if there is an exact match, and an error value of #N/A if there is not. It is now the usual practice to wrap an exact VLOOKUP in an IFERROR function in order to catch the no-match gracefully. Prior to the introduction of IFERROR, no matches were checked with an IF function using the VLOOKUP formula once to check whether there was a match, and once to return the actual match value.

Though initially harder to master, deusxmach1na's proposed solution is a variation on a powerful set of alternatives to VLOOKUP that can be used to return values for a column or list to the left of the lookup column, expanded to handle cases where an exact match on more than one criterion is needed, or modified to incorporate OR as well as AND match conditions among multiple criteria.

Repeating kcamara's chosen solution, the VLOOKUP formula for this problem would be:

   =VLOOKUP(A1,Sheet1!A$1:B$600,2,FALSE)

How can I combine multiple nested Substitute functions in Excel?

Thanks for the idea of breaking down a formula Werner!

Using Alt+Enter allows one to put each bit of a complex substitute formula on separate lines: they become easier to follow and automatically line themselves up when Enter is pressed.

Just make sure you have enough end statements to match the number of substitute( lines either side of the cell reference.

As in this example:

=
substitute(
substitute(
substitute(
substitute(
B11
,"(","")
,")","")
,"[","")
,"]","")

becomes:

=
SUBSTITUTE(
SUBSTITUTE(
SUBSTITUTE(
SUBSTITUTE(B12,"(",""),")",""),"[",""),"]","")

which works fine as is, but one can always delete the extra paragraphs manually:

=SUBSTITUTE(SUBSTITUTE(SUBSTITUTE(SUBSTITUTE(B12,"(",""),")",""),"[",""),"]","")

Name > substitute()

[American Samoa] > American Samoa

Shortcut to Apply a Formula to an Entire Column in Excel

Try double-clicking on the bottom right hand corner of the cell (ie on the box that you would otherwise drag).

Sum values from multiple rows using vlookup or index/match functions

You should use Ctrl+shift+enter when using the =SUM(VLOOKUP(A9,A1:D5,{2,3,4,},FALSE)) that results in {=SUM(VLOOKUP(A9,A1:D5,{2,3,4,},FALSE))} en also works.

In excel how do I reference the current row but a specific column?

If you dont want to hard-code the cell addresses you can use the ROW() function.

eg: =AVERAGE(INDIRECT("A" & ROW()), INDIRECT("C" & ROW()))

Its probably not the best way to do it though! Using Auto-Fill and static columns like @JaiGovindani suggests would be much better.

A formula to copy the values from a formula to another column

Use =concatenate(). Concatenate is generally used to combine the words of several cells into one, but if you only input one cell it will return that value. There are other methods, but I find this is the best because it is the only method that works when a formula, whose value you wish to return, is in a merged cell.

Combining COUNT IF AND VLOOK UP EXCEL

You can combine this all into one formula, but you need to use a regular IF first to find out if the VLOOKUP came back with something, then use your COUNTIF if it did.

=IF(ISERROR(VLOOKUP(B1,Sheet2!A1:A9,1,FALSE)),"Not there",COUNTIF(Sheet2!A1:A9,B1))

In this case, Sheet2-A1:A9 is the range I was searching, and Sheet1-B1 had the value I was looking for ("To retire" in your case).

How to create strings containing double quotes in Excel formulas?

Concatenate " as a ceparate cell:

    A |   B   | C | D
1   " | text  | " | =CONCATENATE(A1; B1; C1);

D1 displays "text"

Get content of a cell given the row and column numbers

Try =index(ARRAY, ROW, COLUMN)

where: Array: select the whole sheet Row, Column: Your row and column references

That should be easier to understand to those looking at the formula.

Set formula to a range of cells

Use FormulaR1C1:

Cells((1,3),(10,3)).FormulaR1C1 = "=RC[-2]+RC[-1]"

Unlike Formula, FormulaR1C1 has relative referencing.

Excel formula to search if all cells in a range read "True", if not, then show "False"

As it appears you have the values as text, and not the numeric True/False, then you can use either COUNTIF or SUMPRODUCT

=IF(SUMPRODUCT(--(A2:D2="False")),"False","True")
=IF(COUNTIF(A3:D3,"False*"),"False","True")

Conditional formatting using AND() function

I was having the same problem with the AND() breaking the conditional formatting. I just happened to try treating the AND as multiplication, and it works! Remove the AND() function and just multiply your arguments. Excel will treat the booleans as 1 for true and 0 for false. I just tested this formula and it seems to work.

=(INDIRECT(ADDRESS(4,COLUMN()))>=INDIRECT(ADDRESS(ROW(),4)))*(INDIRECT(ADDRESS(4,COLUMN()))<=INDIRECT(ADDRESS(ROW(),5)))

How can I count the rows with data in an Excel sheet?

With formulas, what you can do is:

  • in a new column (say col D - cell D2), add =COUNTA(A2:C2)
  • drag this formula till the end of your data (say cell D4 in our example)
  • add a last formula to sum it up (e.g in cell D5): =SUM(D2:D4)

If Cell Starts with Text String... Formula

I know this is a really old post, but I found it in searching for a solution to the same problem. I don't want a nested if-statement, and Switch is apparently newer than the version of Excel I'm using. I figured out what was going wrong with my code, so I figured I'd share here in case it helps someone else.

I remembered that VLOOKUP requires the source table to be sorted alphabetically/numerically for it to work. I was initially trying to do this...

=LOOKUP(LOWER(LEFT($T$3, 1)),  {"s","l","m"}, {-1,1,0})

and it started working when I did this...

=LOOKUP(LOWER(LEFT($T$3, 1)),  {"l","m","s"}, {1,0,-1})

I was initially thinking the last value might turn out to be a default, so I wanted the zero at the last place. That doesn't seem to be the behavior anyway, so I just put the possible matches in order, and it worked.

Edit: As a final note, I see that the example in the original post has letters in alphabetical order, but I imagine the real use case might have been different if the error was happening and the letters A, B, and C were just examples.

Excel- compare two cell from different sheet, if true copy value from other cell

In your destination field you want to use VLOOKUP like so:

=VLOOKUP(Sheet1!A1:A100,Sheet2!A1:F100,6,FALSE)

VLOOKUP Arguments:

  1. The set fields you want to lookup.
  2. The table range you want to lookup up your value against. The first column of your defined table should be the column you want compared against your lookup field. The table range should also contain the value you want to display (Column F).
  3. This defines what field you want to display upon a match.
  4. FALSE tells VLOOKUP to do an exact match.

How does one sum only those rows in excel not filtered out?

You need to use the SUBTOTAL function. The SUBTOTAL function ignores rows that have been excluded by a filter.

The formula would look like this:

=SUBTOTAL(9,B1:B20)

The function number 9, tells it to use the SUM function on the data range B1:B20.

If you are 'filtering' by hiding rows, the function number should be updated to 109.

=SUBTOTAL(109,B1:B20)

The function number 109 is for the SUM function as well, but hidden rows are ignored.

How to enter a formula into a cell using VBA?

I would do it like this:

Worksheets("EmployeeCosts").Range("B" & var1a).Formula = _
Replace("=SUM(H5:H{SOME_VAR})","{SOME_VAR}",var1a)

In case you have some more complex formula it will be handy

How can I perform a reverse string search in Excel without using VBA?

=LEFT(A1,FIND(IF(
 ISERROR(
  FIND("_",A1)
 ),A1,RIGHT(A1,
  LEN(A1)-FIND("~",
   SUBSTITUTE(A1,"_","~",
    LEN(A1)-LEN(SUBSTITUTE(A1,"_",""))
   )
  )
 )
),A1,1)-2)

Excel formula is only showing the formula rather than the value within the cell in Office 2010

Make sure the format of the cell is set to 'general' not 'text' (right click on cell and choose format cell)

Formula should look something like this:

=1+2

Getting unique values in Excel by using formulas only

Solution

I created a function in VBA for you, so you can do this now in an easy way.
Create a VBA code module (macro) as you can see in this tutorial.

  1. Press Alt+F11
  2. Click to Module in Insert.
  3. Paste code.
  4. If Excel says that your file format is not macro friendly than save it as Excel Macro-Enabled in Save As.

Source code

Function listUnique(rng As Range) As Variant
    Dim row As Range
    Dim elements() As String
    Dim elementSize As Integer
    Dim newElement As Boolean
    Dim i As Integer
    Dim distance As Integer
    Dim result As String

    elementSize = 0
    newElement = True

    For Each row In rng.Rows
        If row.Value <> "" Then
            newElement = True
            For i = 1 To elementSize Step 1
                If elements(i - 1) = row.Value Then
                    newElement = False
                End If
            Next i
            If newElement Then
                elementSize = elementSize + 1
                ReDim Preserve elements(elementSize - 1)
                elements(elementSize - 1) = row.Value
            End If
        End If
    Next

    distance = Range(Application.Caller.Address).row - rng.row

    If distance < elementSize Then
        result = elements(distance)
        listUnique = result
    Else
        listUnique = ""
    End If
End Function

Usage

Just enter =listUnique(range) to a cell. The only parameter is range that is an ordinary Excel range. For example: A$1:A$28 or H$8:H$30.

Conditions

  • The range must be a column.
  • The first cell where you call the function must be in the same row where the range starts.

Example

Regular case

  1. Enter data and call function.
    Enter data and call function
  2. Grow it.
    Grow it
  3. Voilà.
    Voilà

Empty cell case

It works in columns that have empty cells in them. Also the function outputs nothing (not errors) if you overwind the cells (calling the function) into places where should be no output, as I did it in the previous example's "2. Grow it" part.

Empty cell case

Return Max Value of range that is determined by an Index & Match lookup

You can easily change the match-type to 1 when you are looking for the greatest value or to -1 when looking for the smallest value.

Declaring variables in Excel Cells

I also just found out how to do this with the Excel Name Manager (Formulas > Defined Names Section > Name Manager).

You can define a variable that doesn't have to "live" within a cell and then you can use it in formulas.

Excel Name Manager

How to add one day to a date?

I will show you how we can do it in Java 8. Here you go:

public class DemoDate {
    public static void main(String[] args) {
        LocalDate today = LocalDate.now();
        System.out.println("Current date: " + today);

        //add 1 day to the current date
        LocalDate date1Day = today.plus(1, ChronoUnit.DAYS);
        System.out.println("Date After 1 day : " + date1Day);
    }
}

The output:

Current date: 2016-08-15
Date After 1 day : 2016-08-16

How to call a function after a div is ready?

inside your <div></div> element you can call the $(document).ready(function(){}); execute a command, something like

<div id="div1">
    <script>
        $(document).ready(function(){
         //do something
        });
    </script>
</div>

and you can do the same to other divs that you have. this was suitable if you loading your div via partial view

Multiple submit buttons on HTML form – designate one button as default

My suggestion is don't fight this behaviour. You can effectively alter the order using floats. For example:

<p id="buttons">
<input type="submit" name="next" value="Next">
<input type="submit" name="prev" value="Previous">
</p>

with:

#buttons { overflow: hidden; }
#buttons input { float: right; }

will effectively reverse the order and thus the "Next" button will be the value triggered by hitting enter.

This kind of technique will cover many circumstances without having to resort to more hacky JavaScript methods.

How can I get a specific parameter from location.search?

A Simple One-Line Solution:

let query = Object.assign.apply(null, location.search.slice(1).split('&').map(entry => ({ [entry.split('=')[0]]: entry.split('=')[1] })));

Expanded & Explained:

// define variable
let query;

// fetch source query
query = location.search;

// skip past the '?' delimiter
query = query.slice(1);

// split source query by entry delimiter
query = query.split('&');

// replace each query entry with an object containing the query entry
query = query.map((entry) => {

   // get query entry key
   let key = entry.split('=')[0];

   // get query entry value
   let value = entry.split('=')[1];

   // define query object
   let container = {};

   // add query entry to object
   container[key] = value;

   // return query object
   return container;
});

// merge all query objects
query = Object.assign.apply(null, query);

Does JavaScript have a built in stringbuilder class?

When I find myself doing a lot of string concatenation in JavaScript, I start looking for templating. Handlebars.js works quite well keeping the HTML and JavaScript more readable. http://handlebarsjs.com

jQuery click anywhere in the page except on 1 div

I know that this question has been answered, And all the answers are nice. But I wanted to add my two cents to this question for people who have similar (but not exactly the same) problem.

In a more general way, we can do something like this:

$('body').click(function(evt){    
    if(!$(evt.target).is('#menu_content')) {
        //event handling code
    }
});

This way we can handle not only events fired by anything except element with id menu_content but also events that are fired by anything except any element that we can select using CSS selectors.

For instance in the following code snippet I am getting events fired by any element except all <li> elements which are descendants of div element with id myNavbar.

$('body').click(function(evt){    
    if(!$(evt.target).is('div#myNavbar li')) {
        //event handling code
    }
});

How can I add raw data body to an axios request?

You can use the below for passing the raw text.

axios.post(
        baseUrl + 'applications/' + appName + '/dataexport/plantypes' + plan, 
        body, 
        {
            headers: { 
                'Authorization': 'Basic xxxxxxxxxxxxxxxxxxx',
                'Content-Type' : 'text/plain' 
            }
        }
).then(response => {
    this.setState({data:response.data});
    console.log(this.state.data);
});

Just have your raw text within body or pass it directly within quotes as 'raw text to be sent' in place of body.

The signature of the axios post is axios.post(url[, data[, config]]), so the data is where you pass your request body.

Cannot find control with name: formControlName in angular reactive form

I tried to generate a form dynamically because the amount of questions depend on an object and for me the error was fixed when I added ngDefaultControl to my mat-form-field.

    <form [formGroup]="questionsForm">
        <ng-container *ngFor="let question of questions">
            <mat-form-field [formControlName]="question.id" ngDefaultControl>
                <mat-label>{{question.questionContent}}</mat-label>
                <textarea matInput rows="3" required></textarea>
            </mat-form-field>
        </ng-container>
        <button mat-raised-button (click)="sendFeedback()">Submit all questions</button>
    </form>

In sendFeedback() I get the value from my dynamic form by selecting the formgroup's value as such

  sendFeedbackAsAgent():void {
    if (this.questionsForm.valid) {
      console.log(this.questionsForm.value)
    }
  }

Swift - How to convert String to Double

SWIFT 3

To clear, nowadays there is a default method:

public init?(_ text: String)` of `Double` class.

It can be used for all classes.

let c = Double("-1.0")
let f = Double("0x1c.6")
let i = Double("inf")

, etc.

How to launch a Google Chrome Tab with specific URL using C#

As a simplification to chrfin's response, since Chrome should be on the run path if installed, you could just call:

Process.Start("chrome.exe", "http://www.YourUrl.com");

This seem to work as expected for me, opening a new tab if Chrome is already open.

How to get screen width without (minus) scrollbar?

I experienced a similar problem and doing width:100%; solved it for me. I came to this solution after trying an answer in this question and realizing that the very nature of an <iframe> will make these javascript measurement tools inaccurate without using some complex function. Doing 100% is a simple way to take care of it in an iframe. I don't know about your issue since I'm not sure of what HTML elements you are manipulating.

How can I pad a String in Java?

public static String LPad(String str, Integer length, char car) {
  return (str + String.format("%" + length + "s", "").replace(" ", String.valueOf(car))).substring(0, length);
}

public static String RPad(String str, Integer length, char car) {
  return (String.format("%" + length + "s", "").replace(" ", String.valueOf(car)) + str).substring(str.length(), length + str.length());
}

LPad("Hi", 10, 'R') //gives "RRRRRRRRHi"
RPad("Hi", 10, 'R') //gives "HiRRRRRRRR"
RPad("Hi", 10, ' ') //gives "Hi        "
RPad("Hi", 1, ' ')  //gives "H"
//etc...

Redirecting to a new page after successful login

Just add the following code after the final message you give using PHP code

Print'window.location.assign("index.php")

Git: Could not resolve host github.com error while cloning remote repository in git

Edge case here but I tried (almost) all of the above answers above on VirtualBox and nothing was doing it but then closing not only the VirtualBoxVM but good ole VirtualBox itself and restarting the program itself did the trick without 0 complaint.

Hope that can help ~0.1% of queriers : )

Add common prefix to all cells in Excel

  1. Enter the function of = CONCATENATE("X",A1) in one cell other than A say D
  2. Click the Cell D1, and drag the fill handle across the range that you want to fill.All the cells should have been added the specific prefix text.

You can see the changes made to the repective cells.

Overwriting my local branch with remote branch

Your local branch likely has modifications to it you want to discard. To do this, you'll need to use git reset to reset the branch head to the last spot that you diverged from the upstream repo's branch. Use git branch -v to find the sha1 id of the upstream branch, and reset your branch it it using git reset SHA1ID. Then you should be able to do a git checkout to discard the changes it left in your directory.

Note: always do this on a backed-up repo. That way you can assure you're self it worked right. Or if it didn't, you have a backup to revert to.

PG COPY error: invalid input syntax for integer

this ought to work without you modifying the source csv file:

alter table people alter column age type text;
copy people from '/tmp/people.csv' with csv;

RestSharp JSON Parameter Posting

You might need to Deserialize your anonymous JSON type from the request body.

var jsonBody = HttpContext.Request.Content.ReadAsStringAsync().Result;
ScoreInputModel myDeserializedClass = JsonConvert.DeserializeObject<ScoreInputModel>(jsonBody);

Dealing with nginx 400 "The plain HTTP request was sent to HTTPS port" error

Actually you can do this with:

ssl off; 

This solved my problem in using nginxvhosts; now I am able to use both SSL and plain HTTP. Works even with combined ports.

How to Update Date and Time of Raspberry Pi With out Internet

Thanks for the replies.
What I did was,
1. I install meinberg ntp software application on windows 7 pc. (softros ntp server is also possible.)
2. change raspberry pi ntp.conf file (for auto update date and time)

server xxx.xxx.xxx.xxx iburst
server 1.debian.pool.ntp.org iburst
server 2.debian.pool.ntp.org iburst
server 3.debian.pool.ntp.org iburst

3. If you want to make sure that date and time update at startup run this python script in rpi,

import os

try:
    client = ntplib.NTPClient()
    response = client.request('xxx.xxx.xxx.xxx', version=4)
    print "===================================="
    print "Offset : "+str(response.offset)
    print "Version : "+str(response.version)
    print "Date Time : "+str(ctime(response.tx_time))
    print "Leap : "+str(ntplib.leap_to_text(response.leap))
    print "Root Delay : "+str(response.root_delay)
    print "Ref Id : "+str(ntplib.ref_id_to_text(response.ref_id))
    os.system("sudo date -s '"+str(ctime(response.tx_time))+"'")
    print "===================================="
except:
    os.system("sudo date")
    print "NTP Server Down Date Time NOT Set At The Startup"
    pass

I found more info in raspberry pi forum.

How to use the "required" attribute with a "radio" input field

You can use this code snippet ...

<html>
  <body>
     <form>
          <input type="radio" name="color" value="black" required />
          <input type="radio" name="color" value="white" />
          <input type="submit" value="Submit" />
    </form>
  </body>
</html>

Specify "required" keyword in one of the select statements. If you want to change the default way of its appearance. You can follow these steps. This is just for extra info if you have any intention to modify the default behavior.

Add the following into you .css file.

/* style all elements with a required attribute */
:required {
  background: red;
}

For more information you can refer following URL.

https://css-tricks.com/almanac/selectors/r/required/

How to get the current date and time

Java has always got inadequate support for the date and time use cases. For example, the existing classes (such as java.util.Date and SimpleDateFormatter) aren’t thread-safe which can lead to concurrency issues. Also there are certain flaws in API. For example, years in java.util.Date start at 1900, months start at 1, and days start at 0—not very intuitive. These issues led to popularity of third-party date and time libraries, such as Joda-Time. To address a new date and time API is designed for Java SE 8.

LocalDateTime timePoint = LocalDateTime.now();
System.out.println(timePoint);

As per doc:

The method now() returns the current date-time using the system clock and default time-zone, not null. It obtains the current date-time from the system clock in the default time-zone. This will query the system clock in the default time-zone to obtain the current date-time. Using this method will prevent the ability to use an alternate clock for testing because the clock is hard-coded.

Flask Error: "Method Not Allowed The method is not allowed for the requested URL"

I had the same problem, and my solving was to replace :

return redirect(url_for('index'))

with

return render_template('indexo.html',data=Todos.query.all())

in my POST and DELETE route.

How do I split a string so I can access item x?

You can leverage a Number table to do the string parsing.

Create a physical numbers table:

    create table dbo.Numbers (N int primary key);
    insert into dbo.Numbers
        select top 1000 row_number() over(order by number) from master..spt_values
    go

Create test table with 1000000 rows

    create table #yak (i int identity(1,1) primary key, array varchar(50))

    insert into #yak(array)
        select 'a,b,c' from dbo.Numbers n cross join dbo.Numbers nn
    go

Create the function

    create function [dbo].[ufn_ParseArray]
        (   @Input      nvarchar(4000), 
            @Delimiter  char(1) = ',',
            @BaseIdent  int
        )
    returns table as
    return  
        (   select  row_number() over (order by n asc) + (@BaseIdent - 1) [i],
                    substring(@Input, n, charindex(@Delimiter, @Input + @Delimiter, n) - n) s
            from    dbo.Numbers
            where   n <= convert(int, len(@Input)) and
                    substring(@Delimiter + @Input, n, 1) = @Delimiter
        )
    go

Usage (outputs 3mil rows in 40s on my laptop)

    select * 
    from #yak 
    cross apply dbo.ufn_ParseArray(array, ',', 1)

cleanup

    drop table dbo.Numbers;
    drop function  [dbo].[ufn_ParseArray]

Performance here is not amazing, but calling a function over a million row table is not the best idea. If performing a string split over many rows I would avoid the function.

How to use HTTP GET in PowerShell?

In PowerShell v3, have a look at the Invoke-WebRequest and Invoke-RestMethod e.g.:

$msg = Read-Host -Prompt "Enter message"
$encmsg = [System.Web.HttpUtility]::UrlEncode($msg)
Invoke-WebRequest -Uri "http://smsserver/SNSManager/msgSend.jsp?uid&to=smartsms:*+001XXXXXX&msg=$encmsg&encoding=windows-1255"

How to detect responsive breakpoints of Twitter Bootstrap 3 using JavaScript?

Since bootstrap 4 will be out soon I thought I would share a function that supports it (xl is now a thing) and performs minimal jQuery to get the job done.

/**
 * Get the Bootstrap device size
 * @returns {string|boolean} xs|sm|md|lg|xl on success, otherwise false if Bootstrap is not working or installed
 */
function findBootstrapEnvironment() {
    var environments = ['xs', 'sm', 'md', 'lg', 'xl'];
    var $el = $('<span />');
    $el.appendTo($('body'));
    for (var i = environments.length - 1; i >= 0; i--) {
        var env = environments[i];
        $el.addClass('hidden-'+env);
        if ($el.is(':hidden')) {
            $el.remove();
            return env;
        }
    }
    $el.remove();
    return false;
}

SELECT * FROM X WHERE id IN (...) with Dapper ORM

Dapper supports this directly. For example...

string sql = "SELECT * FROM SomeTable WHERE id IN @ids"
var results = conn.Query(sql, new { ids = new[] { 1, 2, 3, 4, 5 }});

Fastest way to ping a network range and return responsive hosts?

You should use NMAP:

nmap -T5 -sP 192.168.0.0-255

Mongodb: failed to connect to server on first connect

I connected to a VPN and it worked. I was using school's WiFi.

Angular2: Cannot read property 'name' of undefined

The variable selectedHero is null in the template so you cannot bind selectedHero.name as is. You need to use the elvis operator ?. for this case:

<input [ngModel]="selectedHero?.name" (ngModelChange)="selectedHero.name = $event" />

The separation of the [(ngModel)] into [ngModel] and (ngModelChange) is also needed because you can't assign to an expression that uses the elvis operator.

I also think you mean to use:

<h2>{{selectedHero?.name}} details!</h2>

instead of:

<h2>{{hero.name}} details!</h2>

Get values from label using jQuery

You can use the attr method. For example, if you have a jQuery object called label, you could use this code:

console.log(label.attr("year")); // logs the year
console.log(label.attr("month")); // logs the month

How do you clear a slice in Go?

Setting the slice to nil is the best way to clear a slice. nil slices in go are perfectly well behaved and setting the slice to nil will release the underlying memory to the garbage collector.

See playground

package main

import (
    "fmt"
)

func dump(letters []string) {
    fmt.Println("letters = ", letters)
    fmt.Println(cap(letters))
    fmt.Println(len(letters))
    for i := range letters {
        fmt.Println(i, letters[i])
    }
}

func main() {
    letters := []string{"a", "b", "c", "d"}
    dump(letters)
    // clear the slice
    letters = nil
    dump(letters)
    // add stuff back to it
    letters = append(letters, "e")
    dump(letters)
}

Prints

letters =  [a b c d]
4
4
0 a
1 b
2 c
3 d
letters =  []
0
0
letters =  [e]
1
1
0 e

Note that slices can easily be aliased so that two slices point to the same underlying memory. The setting to nil will remove that aliasing.

This method changes the capacity to zero though.

Using 'make' on OS X

I found the Developer Tools not as readily available as others. In El Capitan, in terminal I just used gcc -v, it then said gcc wasn't available and asked if I wanted to install the command line Apple Developer Tools. No downloading of Xcode required. Terminal session below:

Pauls-MBP:~ paulhillman$ gcc -v
xcode-select: note: no developer tools were found at '/Applications/Xcode.app', requesting install. Choose an option in the dialog to download the command line developer tools.
Pauls-MBP:~ paulhillman$ gcc -v
Configured with: --prefix=/Library/Developer/CommandLineTools/usr --with-gxx-include-dir=/usr/include/c++/4.2.1
Apple LLVM version 7.3.0 (clang-703.0.31)
Target: x86_64-apple-darwin15.6.0
Thread model: posix
InstalledDir: /Library/Developer/CommandLineTools/usr/bin

Best practices for styling HTML emails

I find that image mapping works pretty well. If you have any headers or footers that are images make sure that you apply a bgcolor="fill in the blank" because outlook in most cases wont load the image and you will be left with a transparent header. If you at least designate a color that works with the over all feel of the email it will be less of a shock for the user. Never try and use any styling sheets. Or CSS at all! Just avoid it.

Depending if you're copying content from a word or shared google Doc be sure to (command+F) Find all the (') and (") and replace them within your editing software (especially dreemweaver) because they will show up as code and it's just not good.

ALT is your best friend. use the ALT tag to add in text to all your images. Because odds are they are not going to load right. And that ALT text is what gets people to click the (see images) button. Also define your images Width, Height and make the boarder 0 so you dont get weird lines around your image.

Consider editing all images within Photoshop with a 15px boarder on each side (make background transparent and save as a PNG 24) of image. Sometimes the email clients do not read any padding styles that you apply to the images so it avoids any weird formatting!

Also i found the line under links particularly annoying so if you apply < style="text-decoration:none; color:#whatever color you want here!" > it will remove the line and give you the desired look.

There is alot that can really mess with the over all look and feel.

Making a POST call instead of GET using urllib2

url="https://myserver/post_service"
data["name"] = "joe"
data["age"] = "20"
data_encoded = urllib2.urlencode(data)
print urllib2.urlopen(url + "?" + data_encoded).read()

May be this can help

Increase max_execution_time in PHP?

For increasing execution time and file size, you need to mention below values in your .htaccess file. It will work.

php_value upload_max_filesize 80M
php_value post_max_size 80M
php_value max_input_time 18000
php_value max_execution_time 18000

Run two async tasks in parallel and collect results in .NET 4.5

You should use Task.Delay instead of Sleep for async programming and then use Task.WhenAll to combine the task results. The tasks would run in parallel.

public class Program
    {
        static void Main(string[] args)
        {
            Go();
        }
        public static void Go()
        {
            GoAsync();
            Console.ReadLine();
        }
        public static async void GoAsync()
        {

            Console.WriteLine("Starting");

            var task1 = Sleep(5000);
            var task2 = Sleep(3000);

            int[] result = await Task.WhenAll(task1, task2);

            Console.WriteLine("Slept for a total of " + result.Sum() + " ms");

        }

        private async static Task<int> Sleep(int ms)
        {
            Console.WriteLine("Sleeping for {0} at {1}", ms, Environment.TickCount);
            await Task.Delay(ms);
            Console.WriteLine("Sleeping for {0} finished at {1}", ms, Environment.TickCount);
            return ms;
        }
    }

How can I force WebKit to redraw/repaint to propagate style changes?

I came up here because I needed to redraw scrollbars in Chrome after changing its css.

If someone's having the same problem, I solved it by calling this function:

//Hack to force scroll redraw
function scrollReDraw() {
    $('body').css('overflow', 'hidden').height();
    $('body').css('overflow', 'auto');
}

This method is not the best solution, but it may work with everything, hiding and showing the element that needs to be redraw may solve every problem.

Here is the fiddle where I used it: http://jsfiddle.net/promatik/wZwJz/18/

How to tell if tensorflow is using gpu acceleration from inside python shell?

Ok, first launch an ipython shell from the terminal and import TensorFlow:

$ ipython --pylab
Python 3.6.5 |Anaconda custom (64-bit)| (default, Apr 29 2018, 16:14:56) 
Type 'copyright', 'credits' or 'license' for more information
IPython 6.4.0 -- An enhanced Interactive Python. Type '?' for help.
Using matplotlib backend: Qt5Agg

In [1]: import tensorflow as tf

Now, we can watch the GPU memory usage in a console using the following command:

# realtime update for every 2s
$ watch -n 2 nvidia-smi

Since we've only imported TensorFlow but have not used any GPU yet, the usage stats will be:

tf non-gpu usage

Notice how the GPU memory usage is very less (~ 700MB); Sometimes the GPU memory usage might even be as low as 0 MB.


Now, let's load the GPU in our code. As indicated in tf documentation, do:

In [2]: sess = tf.Session(config=tf.ConfigProto(log_device_placement=True))

Now, the watch stats should show an updated GPU usage memory as below:

tf gpu-watch

Observe now how our Python process from the ipython shell is using ~ 7 GB of the GPU memory.


P.S. You can continue watching these stats as the code is running, to see how intense the GPU usage is over time.

anaconda - path environment variable in windows

You could also just re-install Anaconda, and tick the option add variable to Path.. This will prevent you from making mistakes when editing environment variables. If you make mistakes here, your operating system could start malfunctioning.

js 'types' can only be used in a .ts file - Visual Studio Code using @ts-check

I'm using flow with vscode but had the same problem. I solved it with these steps:

  1. Install the extension Flow Language Support

  2. Disable the built-in TypeScript extension:

    1. Go to Extensions tab
    2. Search for @builtin TypeScript and JavaScript Language Features
    3. Click on Disable

Add Items to Columns in a WPF ListView

Solution With Less XAML and More C#

If you define the ListView in XAML:

<ListView x:Name="listView"/>

Then you can add columns and populate it in C#:

public Window()
{
    // Initialize
    this.InitializeComponent();

    // Add columns
    var gridView = new GridView();
    this.listView.View = gridView;
    gridView.Columns.Add(new GridViewColumn { 
        Header = "Id", DisplayMemberBinding = new Binding("Id") });
    gridView.Columns.Add(new GridViewColumn { 
        Header = "Name", DisplayMemberBinding = new Binding("Name") });

    // Populate list
    this.listView.Items.Add(new MyItem { Id = 1, Name = "David" });
}

See definition of MyItem below.

Solution With More XAML and less C#

However, it's easier to define the columns in XAML (inside the ListView definition):

<ListView x:Name="listView">
    <ListView.View>
        <GridView>
            <GridViewColumn Header="Id" DisplayMemberBinding="{Binding Id}"/>
            <GridViewColumn Header="Name" DisplayMemberBinding="{Binding Name}"/>
        </GridView>
    </ListView.View>
</ListView>

And then just populate the list in C#:

public Window()
{
    // Initialize
    this.InitializeComponent();

    // Populate list
    this.listView.Items.Add(new MyItem { Id = 1, Name = "David" });
}

See definition of MyItem below.

MyItem Definition

MyItem is defined like this:

public class MyItem
{
    public int Id { get; set; }

    public string Name { get; set; }
}

How do I run a bat file in the background from another bat file?

This works on my Windows XP Home installation, the Unix way:

call notepad.exe & 

How to deep merge instead of shallow merge?

Here, straight forward;

a simple solution that works like Object.assign just deep, and works for an array, without any modification.

_x000D_
_x000D_
function deepAssign(target, ...sources) {
  for (source of sources) {
    for (let k in source) {
      let vs = source[k], vt = target[k]
      if (Object(vs) == vs && Object(vt) === vt) {
        target[k] = deepAssign(vt, vs)
        continue
      }
      target[k] = source[k]
    }
  }
  return target
}

x = { a: { a: 1 }, b: [1,2] }
y = { a: { b: 1 }, b: [3] }
z = { c: 3, b: [,,,4] }
x = deepAssign(x, y, z)

console.log(JSON.stringify(x) === JSON.stringify({
  "a": {
    "a": 1,
    "b": 1
  },
  "b": [ 1, 2, null, 4 ],
  "c": 3
}))
_x000D_
_x000D_
_x000D_

Increase permgen space

if you found out that the memory settings were not being used and in order to change the memory settings, I used the tomcat7w or tomcat8w in the \bin folder.Then the following should pop up:

tomcat monitor

Click the Java tab and add the arguments.restart tomcat

AngularJS not detecting Access-Control-Allow-Origin header?

If you guys are having this problem in sails.js just set your cors.js to include Authorization as the allowed header

_x000D_
_x000D_
/***************************************************************************_x000D_
  *                                                                          *_x000D_
  * Which headers should be allowed for CORS requests? This is only used in  *_x000D_
  * response to preflight requests.                                          *_x000D_
  *                                                                          *_x000D_
  ***************************************************************************/_x000D_
_x000D_
  headers: 'Authorization' // this line here
_x000D_
_x000D_
_x000D_

Class has no objects member

Change your linter to - flake8 and problem will go away.

GC overhead limit exceeded

From Java SE 6 HotSpot[tm] Virtual Machine Garbage Collection Tuning

the following

Excessive GC Time and OutOfMemoryError

The concurrent collector will throw an OutOfMemoryError if too much time is being spent in garbage collection: if more than 98% of the total time is spent in garbage collection and less than 2% of the heap is recovered, an OutOfMemoryError will be thrown. This feature is designed to prevent applications from running for an extended period of time while making little or no progress because the heap is too small. If necessary, this feature can be disabled by adding the option -XX:-UseGCOverheadLimit to the command line.

The policy is the same as that in the parallel collector, except that time spent performing concurrent collections is not counted toward the 98% time limit. In other words, only collections performed while the application is stopped count toward excessive GC time. Such collections are typically due to a concurrent mode failure or an explicit collection request (e.g., a call to System.gc()).

in conjunction with a passage further down

One of the most commonly encountered uses of explicit garbage collection occurs with RMIs distributed garbage collection (DGC). Applications using RMI refer to objects in other virtual machines. Garbage cannot be collected in these distributed applications without occasionally collection the local heap, so RMI forces full collections periodically. The frequency of these collections can be controlled with properties. For example,

java -Dsun.rmi.dgc.client.gcInterval=3600000

-Dsun.rmi.dgc.server.gcInterval=3600000 specifies explicit collection once per hour instead of the default rate of once per minute. However, this may also cause some objects to take much longer to be reclaimed. These properties can be set as high as Long.MAX_VALUE to make the time between explicit collections effectively infinite, if there is no desire for an upper bound on the timeliness of DGC activity.

Seems to imply that the evaluation period for determining the 98% is one minute long, but it might be configurable on Sun's JVM with the correct define.

Of course, other interpretations are possible.

When does Git refresh the list of remote branches?

Use git fetch to fetch all latest created branches.

Angular 4: InvalidPipeArgument: '[object Object]' for pipe 'AsyncPipe'

async is used for binding to Observables and Promises, but it seems like you're binding to a regular object. You can just remove both async keywords and it should probably work.

Remove file from SVN repository without deleting local copy

Deleting files and folders

If you want to delete an item from the repository, but keep it locally as an unversioned file/folder, use Extended Context Menu ? Delete (keep local). You have to hold the Shift key while right clicking on the item in the explorer list pane (right pane) in order to see this in the extended context menu.

Delete completely:
right mouse click ? Menu ? Delete

Delete & Keep local:
Shift + right mouse click ? Menu ? Delete

How print out the contents of a HashMap<String, String> in ascending order based on its values?

while (itr.hasNext()) {
    Vehicle vc=(Vehicle) itr.next();
    if(vc.getVehicleType().equalsIgnoreCase(s)) {
        count++;
    }
}

Instagram API: How to get all user media?

What I had to do is (in Javascript) is go through all pages by using a recursive function. It's dangerouse as instagram users could have thousands of pictures i a part from that (so your have to controle it) I use this code: (count parameter I think , doesn't do much)

        instagramLoadDashboard = function(hash)
    {
        code = hash.split('=')[1];

        $('#instagram-pictures .images-list .container').html('').addClass('loading');


        ts = Math.round((new Date()).getTime() / 1000);
        url = 'https://api.instagram.com/v1/users/self/media/recent?count=200&min_timestamp=0&max_timestamp='+ts+'&access_token='+code;

        instagramLoadMediaPage(url, function(){

            galleryHTML = instagramLoadGallery(instagramData);
            //console.log(galleryHTML);
            $('#instagram-pictures .images-list .container').html(galleryHTML).removeClass('loading');
            initImages('#instagram-pictures');

            IGStatus = 'loaded';

        });

    };

    instagramLoadMediaPage = function (url, callback)
    {
        $.ajax({
                url : url,
                dataType : 'jsonp',
                cache : false,
                success:  function(response){

                                        console.log(response);

                                        if(response.code == '400')
                                        {
                                            alert(response.error_message);
                                            return false;
                                        }

                                        if(response.pagination.next_url !== undefined) {
                                            instagramData = instagramData.concat(response.data);
                                            return instagramLoadMediaPage(response.pagination.next_url,callback);
                                        }

                                        instagramData = instagramData.concat(response.data);
                                        callback.apply();
                                    }
        });
    };

    instagramLoadGallery = function(images)
    {
        galleryHTML ='<ul>';

        for(var i=0;i<images.length;i++)
        {
            galleryHTML += '<li><img src="'+images[i].images.thumbnail.url+'" width="120" id="instagram-'+images[i].id+' data-type="instagram" data-source="'+images[i].images.standard_resolution.url+'" class="image"/></li>';

        }

        galleryHTML +='</ul>';

        return galleryHTML;
    };

There some stuff related to print out a gallery of picture.

ERROR : [Microsoft][ODBC Driver Manager] Data source name not found and no default driver specified

For anyone coming to this latterly, I was having this problem over a Windows network, and offer an additional thing to check:

Python script connecting would work from commandline on my (linux) machine, but some users had problems connecting - that it worked from CLI suggested the DSN and credentials were right. The issue for us was that the group security policy required the ODBC credentials to be set on every machine. Once we added that (for some reason, the user had three of the four ODBC credentials they needed for our various systems), they were able to connect.

You can of course do that at group level, but as it was a simple omission on the part of one machine, I did it in Control Panel > ODBC Drivers > New

Is embedding background image data into CSS as Base64 good or bad practice?

I disagree with the recommendation to create separate CSS files for non-editorial images.

Assuming the images are for UI purposes, it's presentation layer styling, and as mentioned above, if you're doing mobile UI's its definitely a good idea to keep all styling in a single file so it can be cached once.

How to repair COMException error 80040154?

To find the DLL, go to your 64-bit machine and open the registry. Find the key called HKEY_CLASSES_ROOT\CLSID\{681EF637-F129-4AE9-94BB-618937E3F6B6}\InprocServer32. This key will have the filename of the DLL as its default value.

If you solved the problem on your 64-bit machine by recompiling your project for x86, then you'll need to look in the 32-bit portion of the registry instead of in the normal place. This is HKEY_LOCAL_MACHINE\SOFTWARE\Wow6432Node\Classes\CLSID\{681EF637-F129-4AE9-94BB-618937E3F6B6}\InprocServer32.

If the DLL is built for 32 bits then you can use it directly on your 32-bit machine. If it's built for 64 bits then you'll have to contact the vendor and get a 32-bit version from them.

When you have the DLL, register it by running c:\windows\system32\regsvr32.exe.

Postgres user does not exist?

For me This was the solution on macOS ReInstall the psql

brew install postgres

Start PostgreSQL server

pg_ctl -D /usr/local/var/postgres start

Initialize DB

initdb /usr/local/var/postgres

If this command throws an error the rm the old database file and re-run the above command

rm -r /usr/local/var/postgres

Create a new database

createdb postgres_test
psql -W postegres_test

You will be logged into this db and can create a user in here to login

Add & delete view from Layout

For changing visibility:

predictbtn.setVisibility(View.INVISIBLE);

For removing:

predictbtn.setVisibility(View.GONE);

CSS: center element within a <div> element

Create a new div element for your element to center, then add a class specifically for centering that element like this

<div id="myNewElement">
    <div class="centered">
        <input type="button" value="My Centered Button"/>
    </div>
</div>

Css

.centered{
    text-align:center;
}

How to extract year and month from date in PostgreSQL without using to_char() function?

date_part(text, timestamp)

e.g.

date_part('month', timestamp '2001-02-16 20:38:40'),
date_part('year', timestamp '2001-02-16 20:38:40') 

http://www.postgresql.org/docs/8.0/interactive/functions-datetime.html

Why is @font-face throwing a 404 error on woff files?

If you are using CodeIgniter under IIS7 :

In your web.config file, add woff to the pattern

<rule name="Rewrite CI Index">
  <match url=".*" />
    <conditions>
      <add input="{REQUEST_FILENAME}" pattern="css|js|jpg|jpeg|png|gif|ico|htm|html|woff" negate="true" />
    </conditions>
    <action type="Rewrite" url="index.php/{R:0}" />
 </rule>

Hope it helps !

Sending a mail from a linux shell script

If both exim and ssmtp are running, you may enter into troubles. So if you just want to run a simple MTA, just to have a simple smtp client to send email notifications for insistance, you shall purge the eventually preinstalled MTA like exim or postfix first and reinstall ssmtp.

Then it's quite straight forward, configuring only 2 files (revaliases and ssmtp.conf) - See ssmtp doc - , and usage in your bash or bourne script is like :

#!/bin/sh  
SUBJECT=$1  
RECEIVER=$2  
TEXT=$3  

SERVER_NAME=$HOSTNAME  
SENDER=$(whoami)  
USER="noreply"

[[ -z $1 ]] && SUBJECT="Notification from $SENDER on server $SERVER_NAME"  
[[ -z $2 ]] && RECEIVER="another_configured_email_address"   
[[ -z $3 ]] && TEXT="no text content"  

MAIL_TXT="Subject: $SUBJECT\nFrom: $SENDER\nTo: $RECEIVER\n\n$TEXT"  
echo -e $MAIL_TXT | sendmail -t  
exit $?  

Obviously do not forget to open your firewall output to the smtp port (25).

How to change the background color of a UIButton while it's highlighted?

Try this !!!!

For TouchedDown Event set One color and for TouchUpInside set the other.

- (IBAction)touchedDown:(id)sender {
    NSLog(@"Touched Down");
    btn1.backgroundColor=[UIColor redColor];
}

- (IBAction)touchUpInside:(id)sender {
    NSLog(@"TouchUpInside");
    btn1.backgroundColor=[UIColor whiteColor];    
}

How do I delete everything below row X in VBA/Excel?

It sounds like something like the below will suit your needs:

With Sheets("Sheet1")
    .Rows( X & ":" & .Rows.Count).Delete
End With

Where X is a variable that = the row number ( 415 )

removeEventListener on anonymous functions in JavaScript

window.document.onkeydown = function(){};

Set up a scheduled job?

Simple way is to write a custom shell command see Django Documentation and execute it using a cronjob on linux. However i would highly recommend using a message broker like RabbitMQ coupled with celery. Maybe you can have a look at this Tutorial

Does uninstalling a package with "pip" also remove the dependent packages?

You may have a try for https://github.com/cls1991/pef. It will remove package with its all dependencies.

Custom format for time command

From the man page for time:

  1. There may be a shell built-in called time, avoid this by specifying /usr/bin/time
  2. You can provide a format string and one of the format options is elapsed time - e.g. %E

    /usr/bin/time -f'%E' $CMD

Example:

$ /usr/bin/time -f'%E' ls /tmp/mako/
res.py  res.pyc
0:00.01

Read file line by line in PowerShell

The almighty switch works well here:

'one
two
three' > file

$regex = '^t'

switch -regex -file file { 
  $regex { "line is $_" } 
}

Output:

line is two
line is three

How to create a Jar file in Netbeans

I also tried to make an executable jar file that I could run with the following command:

java -jar <jarfile>

After some searching I found the following link:

Packaging and Deploying Desktop Java Applications

I set the project's main class:

  1. Right-click the project's node and choose Properties
  2. Select the Run panel and enter the main class in the Main Class field
  3. Click OK to close the Project Properties dialog box
  4. Clean and build project

Then in the fodler dist the newly created jar should be executable with the command I mentioned above.

View's getWidth() and getHeight() returns 0

The basic problem is, that you have to wait for the drawing phase for the actual measurements (especially with dynamic values like wrap_content or match_parent), but usually this phase hasn't been finished up to onResume(). So you need a workaround for waiting for this phase. There a are different possible solutions to this:

1. Listen to Draw/Layout Events: ViewTreeObserver

A ViewTreeObserver gets fired for different drawing events. Usually the OnGlobalLayoutListener is what you want for getting the measurement, so the code in the listener will be called after the layout phase, so the measurements are ready:

view.getViewTreeObserver().addOnGlobalLayoutListener(new ViewTreeObserver.OnGlobalLayoutListener() {
            @Override
            public void onGlobalLayout() {
                view.getViewTreeObserver().removeOnGlobalLayoutListener(this);
                view.getHeight(); //height is ready
            }
        });

Note: The listener will be immediately removed because otherwise it will fire on every layout event. If you have to support apps SDK Lvl < 16 use this to unregister the listener:

public void removeGlobalOnLayoutListener (ViewTreeObserver.OnGlobalLayoutListener victim)


2. Add a runnable to the layout queue: View.post()

Not very well known and my favourite solution. Basically just use the View's post method with your own runnable. This basically queues your code after the view's measure, layout, etc. as stated by Romain Guy:

The UI event queue will process events in order. After setContentView() is invoked, the event queue will contain a message asking for a relayout, so anything you post to the queue will happen after the layout pass

Example:

final View view=//smth;
...
view.post(new Runnable() {
            @Override
            public void run() {
                view.getHeight(); //height is ready
            }
        });

The advantage over ViewTreeObserver:

  • your code is only executed once and you don't have to disable the Observer after execution which can be a hassle
  • less verbose syntax

References:


3. Overwrite Views's onLayout Method

This is only practical in certain situation when the logic can be encapsulated in the view itself, otherwise this is a quite verbose and cumbersome syntax.

view = new View(this) {
    @Override
    protected void onLayout(boolean changed, int l, int t, int r, int b) {
        super.onLayout(changed, l, t, r, b);
        view.getHeight(); //height is ready
    }
};

Also mind, that onLayout will be called many times, so be considerate what you do in the method, or disable your code after the first time


4. Check if has been through layout phase

If you have code that is executing multiple times while creating the ui you could use the following support v4 lib method:

View viewYouNeedHeightFrom = ...
...
if(ViewCompat.isLaidOut(viewYouNeedHeightFrom)) {
   viewYouNeedHeightFrom.getHeight();
}

Returns true if view has been through at least one layout since it was last attached to or detached from a window.

Additional: Getting staticly defined measurements

If it suffices to just get the statically defined height/width, you can just do this with:

But mind you, that this might be different to the actual width/height after drawing. The javadoc describes the difference in more detail:

The size of a view is expressed with a width and a height. A view actually possess two pairs of width and height values.

The first pair is known as measured width and measured height. These dimensions define how big a view wants to be within its parent (see Layout for more details.) The measured dimensions can be obtained by calling getMeasuredWidth() and getMeasuredHeight().

The second pair is simply known as width and height, or sometimes drawing width and drawing height. These dimensions define the actual size of the view on screen, at drawing time and after layout. These values may, but do not have to, be different from the measured width and height. The width and height can be obtained by calling getWidth() and getHeight().

Python update a key in dict if it doesn't exist

According to the above answers setdefault() method worked for me.

old_attr_name = mydict.setdefault(key, attr_name)
if attr_name != old_attr_name:
    raise RuntimeError(f"Key '{key}' duplication: "
                       f"'{old_attr_name}' and '{attr_name}'.")

Though this solution is not generic. Just suited me in this certain case. The exact solution would be checking for the key first (as was already advised), but with setdefault() we avoid one extra lookup on the dictionary, that is, though small, but still a performance gain.

How to restart remote MySQL server running on Ubuntu linux?

I SSH'ed into my AWS Lightsail wordpress instance, the following worked: sudo /opt/bitnami/ctlscript.sh restart mysql I learnt this here: https://docs.bitnami.com/aws/infrastructure/mysql/administration/control-services/

Bootstrap: How to center align content inside column?

You can do this by adding a div i.e. centerBlock. And give this property in CSS to center the image or any content. Here is the code:

<div class="container">
    <div class="row">
        <div class="col-sm-4 col-md-4 col-lg-4">
            <div class="centerBlock">
                <img class="img-responsive" src="img/some-image.png" title="This image needs to be centered">
            </div>
        </div>
        <div class="col-sm-8 col-md-8 col-lg-8">
            Some content not important at this moment
        </div>
    </div>
</div>


// CSS

.centerBlock {
  display: table;
  margin: auto;
}

Neatest way to remove linebreaks in Perl

In your example, you can just go:

chomp(@lines);

Or:

$_=join("", @lines);
s/[\r\n]+//g;

Or:

@lines = split /[\r\n]+/, join("", @lines);

Using these directly on a file:

perl -e '$_=join("",<>); s/[\r\n]+//g; print' <a.txt |less

perl -e 'chomp(@a=<>);print @a' <a.txt |less

Create an array of integers property in Objective-C

You can put this in your .h file for your class and define it as property, in XCode 7:

@property int  (*stuffILike) [10];

MySQL Trigger: Delete From Table AFTER DELETE

Why not set ON CASCADE DELETE on Foreign Key patron_info.pid?

Subtract days from a DateTime

I've had issues using AddDays(-1).

My solution is TimeSpan.

DateTime.Now - TimeSpan.FromDays(1);

How to get the current date without the time?

Well, you can get just today's date as a DateTime using the Today property:

 DateTime today = DateTime.Today;

or more generally, you can use the Date property. For example, if you wanted the UTC date you could use:

 DateTime dateTime = DateTime.UtcNow.Date;

It's not very clear whether that's what you need or not though... if you're just looking to print the date, you can use:

Console.WriteLine(dateTime.ToString("d"));

or use an explicit format:

Console.WriteLine(dateTime.ToString("dd/MM/yyyy"));

See more about standard and custom date/time format strings. Depending on your situation you may also want to specify the culture.

If you want a more expressive date/time API which allows you to talk about dates separately from times, you might want to look at the Noda Time project which I started. It's not ready for production just yet, but we'd love to hear what you'd like to do with it...

Catching "Maximum request length exceeded"

In IIS 7 and beyond:

web.config file:

<system.webServer>
  <security >
    <requestFiltering>
      <requestLimits maxAllowedContentLength="[Size In Bytes]" />
    </requestFiltering>
  </security>
</system.webServer>

You can then check in code behind, like so:

If FileUpload1.PostedFile.ContentLength > 2097152 Then ' (2097152 = 2 Mb)
  ' Exceeded the 2 Mb limit
  ' Do something
End If

Just make sure the [Size In Bytes] in the web.config is greater than the size of the file you wish to upload then you won't get the 404 error. You can then check the file size in code behind using the ContentLength which would be much better

Show div when radio button selected

I would handle it like so:

$(document).ready(function() {
   $('input[type="radio"]').click(function() {
       if($(this).attr('id') == 'watch-me') {
            $('#show-me').show();           
       }

       else {
            $('#show-me').hide();   
       }
   });
});

Submit Button Image

<INPUT TYPE="image" SRC="images/submit.gif" HEIGHT="30" WIDTH="173" BORDER="0" ALT="Submit Form">
Where the standard submit button has TYPE="submit", we now have TYPE="image". The image type is by default a form submitting button. More simple

How can I get the console logs from the iOS Simulator?

No NSLog or print content will write to system.log, which can be open by Select Simulator -> Debug -> Open System log on Xcode 11.

I figure out a way, write logs into a file and open the xx.log with Terminal.app.Then the logs will present in Terminal.app lively.

I use CocoaLumberjack achieve this.

STEP 1:

Add DDFileLogger DDOSLogger and print logs path. config() should be called when App lunch.

static func config() {
    #if DEBUG
    DDLog.add(DDOSLogger.sharedInstance) // Uses os_log
    let fileLogger: DDFileLogger = DDFileLogger() // File Logger
    fileLogger.rollingFrequency = 60 * 60 * 24 // 24 hours
    fileLogger.logFileManager.maximumNumberOfLogFiles = 7
    DDLog.add(fileLogger)
    DDLogInfo("DEBUG LOG PATH: " + (fileLogger.currentLogFileInfo?.filePath ?? ""))
    #endif
}

STEP 2:

Replace print or NSLog with DDLogXXX.

STEP 3:

$ tail -f {path of log}

Here, message will present in Terminal.app lively.

One thing more. If there is no any message log out, make sure Environment Variables -> OS_ACTIVITY_MODE ISNOT disable.

Regular expression for matching HH:MM time format

Declare

private static final String TIME24HOURS_PATTERN = "([01]?[0-9]|2[0-3]):[0-5][0-9]:[0-5][0-9]";

public boolean validate(final String time) {
    pattern = Pattern.compile(TIME24HOURS_PATTERN);
    matcher = pattern.matcher(time);
    return matcher.matches();
}

This method return "true" when String match with the Regular Expression.

Remove gutter space for a specific div only

Interesting...

Removing the gutter in Twitter Bootstrap's Default grid, that is, 940px wide. And that the default grid has a 940px wide container and has the bootstrap-responsive.css in it's stylesheet.

If I got your question right, this is how I did it...

<!DOCTYPE html>
<html lang="en">
  <head>
    <meta charset="utf-8">
    <title>Stackoverflow Question</title>
    <meta name="viewport" content="width=device-width, initial-scale=1.0">
    <meta name="description" content="">
    <meta name="author" content="">

    <!-- Le styles -->
    <link rel="stylesheet" href="assets/css/bootstrap.css">
    <link rel="stylesheet" href="assets/css/bootstrap-responsive.css">

    <!-- HTML5 shim, for IE6-8 support of HTML5 elements -->
    <!--[if lt IE 9]>
      <script src="assets/js/html5shiv.js"></script>
    <![endif]-->




    <style type="text/css">
        #main_content [class*="span"] {
            margin-left: 0;
            width: 25%;
        }

        @media (min-width: 768px) and (max-width: 979px) {
            #main_content [class*="span"] {
            margin-left: 0;
            width: 25%;
            }
        }


        @media (max-width: 767px) {
            #main_content [class*="span"] {
            margin-left: 0;
            width: 100%;
            }
        }

        @media (max-width: 480px) {
            #main_content [class*="span"] {
            margin-left: 0;
            width: 100%;
            }
        }

        <!-- For Visual Aid Only -->
        .bg1 {
            background-color: #C2C2C2;
        }

        .bg2 {
            background-color: #D2D2D2;
        }
    </style>
  <body>
    <div id="wrap">
        <div class="container">
            <div class="row-fluid">
                <div class="span1 text-center bg1">01</div>
                <div class="span1 text-center bg2">02</div>
                <div class="span1 text-center bg1">03</div>
                <div class="span1 text-center bg2">04</div>
                <div class="span1 text-center bg1">05</div>
                <div class="span1 text-center bg2">06</div>
                <div class="span1 text-center bg1">07</div>
                <div class="span1 text-center bg2">08</div>
                <div class="span1 text-center bg1">09</div>
                <div class="span1 text-center bg2">10</div>
                <div class="span1 text-center bg1">11</div>
                <div class="span1 text-center bg2">12</div>
            </div>



            <div id="main_content">
                <div class="row-fluid">
                    <div class="span3 text-center bg1">1</div>
                    <div class="span3 text-center bg2">2</div>
                    <div class="span3 text-center bg1">3</div>
                    <div class="span3 text-center bg2">4</div>
                </div>
            </div>
        </div><!--/container-->
    </div>
  </body>
</html>

And the result is..

enter image description here

The 4 div span with no gutter will remain spanned for Small tablet landscape (800x600). Anything size smaller than that will collapse the 4 divs and it will be stacked vertically. Of course you will have to tweak it to fit your needs.

How to concatenate a std::string and an int?

There is a function I wrote, which takes the int number as the parameter, and convert it to a string literal. This function is dependent on another function that converts a single digit to its char equivalent:

char intToChar(int num)
{
    if (num < 10 && num >= 0)
    {
        return num + 48;
        //48 is the number that we add to an integer number to have its character equivalent (see the unsigned ASCII table)
    }
    else
    {
        return '*';
    }
}

string intToString(int num)
{
    int digits = 0, process, single;
    string numString;
    process = num;

    // The following process the number of digits in num
    while (process != 0)
    {
        single  = process % 10; // 'single' now holds the rightmost portion of the int
        process = (process - single)/10;
        // Take out the rightmost number of the int (it's a zero in this portion of the int), then divide it by 10
        // The above combination eliminates the rightmost portion of the int
        digits ++;
    }

    process = num;

    // Fill the numString with '*' times digits
    for (int i = 0; i < digits; i++)
    {
        numString += '*';
    }


    for (int i = digits-1; i >= 0; i--)
    {
        single = process % 10;
        numString[i] = intToChar ( single);
        process = (process - single) / 10;
    }

    return numString;
}

Application not picking up .css file (flask/python)

I have read multiple threads and none of them fixed the issue that people are describing and I have experienced too.

I have even tried to move away from conda and use pip, to upgrade to python 3.7, i have tried all coding proposed and none of them fixed.

And here is why (the problem):

by default python/flask search the static and the template in a folder structure like:

/Users/username/folder_one/folder_two/ProjectName/src/app_name/<static>
 and 
/Users/username/folder_one/folder_two/ProjectName/src/app_name/<template>

you can verify by yourself using the debugger on Pycharm (or anything else) and check the values on the app (app = Flask(name)) and search for teamplate_folder and static_folder

in order to fix this, you have to specify the values when creating the app something like this:

TEMPLATE_DIR = os.path.abspath('../templates')
STATIC_DIR = os.path.abspath('../static')

# app = Flask(__name__) # to make the app run without any
app = Flask(__name__, template_folder=TEMPLATE_DIR, static_folder=STATIC_DIR)

the path TEMPLATE_DIR and STATIC_DIR depend on where the file app is located. in my case, see the picture, it was located within a folder under src.

you can change the template and static folders as you wish and register on the app = Flask...

In truth, I have started experiencing the problem when messing around with folder and at times worked at times not. this fixes the problem once and for all

the html code looks like this:

<link href="{{ url_for('static', filename='libraries/css/bootstrap.css') }}" rel="stylesheet" type="text/css" >

This the code

Here the structure of the folders

Node.js global variables

You can use global like so:

global._ = require('underscore')

PostgreSQL: FOREIGN KEY/ON DELETE CASCADE

PostgreSQL Forging Key DELETE, UPDATE CASCADE

CREATE TABLE apps_user(
  user_id SERIAL PRIMARY KEY,
  username character varying(30),
  userpass character varying(50),
  created_on DATE
);

CREATE TABLE apps_profile(
    pro_id SERIAL PRIMARY KEY,
    user_id INT4 REFERENCES apps_user(user_id) ON DELETE CASCADE ON UPDATE CASCADE,
    firstname VARCHAR(30),
    lastname VARCHAR(50),
    email VARCHAR UNIQUE,
    dob DATE
);

How can I return the difference between two lists?

With Stream API you can do something like this:

List<String> aWithoutB = a.stream()
    .filter(element -> !b.contains(element))
    .collect(Collectors.toList());

List<String> bWithoutA = b.stream()
    .filter(element -> !a.contains(element))
    .collect(Collectors.toList());

SQL query, store result of SELECT in local variable

You can create table variables:

DECLARE @result1 TABLE (a INT, b INT, c INT)

INSERT INTO @result1
SELECT a, b, c
FROM table1

SELECT a AS val FROM @result1
UNION
SELECT b AS val FROM @result1
UNION
SELECT c AS val FROM @result1

This should be fine for what you need.

Is it safe to clean docker/overlay2/

Backgroud

The blame for the issue can be split between our misconfiguration of container volumes, and a problem with docker leaking (failing to release) temporary data written to these volumes. We should be mapping (either to host folders or other persistent storage claims) all of out container's temporary / logs / scratch folders where our apps write frequently and/or heavily. Docker does not take responsibility for the cleanup of all automatically created so-called EmptyDirs located by default in /var/lib/docker/overlay2/*/diff/*. Contents of these "non-persistent" folders should be purged automatically by docker after container is stopped, but apparently are not (they may be even impossible to purge from the host side if the container is still running - and it can be running for months at a time).

Workaround

A workaround requires careful manual cleanup, and while already described elsewhere, you still may find some hints from my case study, which I tried to make as instructive and generalizable as possible.

So what happened is the culprit app (in my case clair-scanner) managed to write over a few months hundreds of gigs of data to the /diff/tmp subfolder of docker's overlay2

du -sch /var/lib/docker/overlay2/<long random folder name seen as bloated in df -haT>/diff/tmp

271G total

So as all those subfolders in /diff/tmp were pretty self-explanatory (all were of the form clair-scanner-* and had obsolete creation dates), I stopped the associated container (docker stop clair) and carefully removed these obsolete subfolders from diff/tmp, starting prudently with a single (oldest) one, and testing the impact on docker engine (which did require restart [systemctl restart docker] to reclaim disk space):

rm -rf $(ls -at /var/lib/docker/overlay2/<long random folder name seen as bloated in df -haT>/diff/tmp | grep clair-scanner | tail -1)

I reclaimed hundreds of gigs of disk space without the need to re-install docker or purge its entire folders. All running containers did have to be stopped at one point, because docker daemon restart was required to reclaim disk space, so make sure first your failover containers are running correctly on an/other node/s). I wish though that the docker prune command could cover the obsolete /diff/tmp (or even /diff/*) data as well (via yet another switch).

It's a 3-year-old issue now, you can read its rich and colorful history on Docker forums, where a variant aimed at application logs of the above solution was proposed in 2019 and seems to have worked in several setups: https://forums.docker.com/t/some-way-to-clean-up-identify-contents-of-var-lib-docker-overlay/30604

Delete from two tables in one query

You should either create a FOREIGN KEY with ON DELETE CASCADE:

ALTER TABLE usersmessages
ADD CONSTRAINT fk_usermessages_messageid
FOREIGN KEY (messageid)
REFERENCES messages (messageid)
ON DELETE CASCADE

, or do it using two queries in a transaction:

START TRANSACTION;;

DELETE
FROM    usermessages
WHERE   messageid = 1

DELETE
FROM    messages
WHERE   messageid = 1;

COMMIT;

Transaction affects only InnoDB tables, though.

Use of REPLACE in SQL Query for newline/ carriage return characters

There are probably embedded tabs (CHAR(9)) etc. as well. You can find out what other characters you need to replace (we have no idea what your goal is) with something like this:

DECLARE @var NVARCHAR(255), @i INT;

SET @i = 1;

SELECT @var = AccountType FROM dbo.Account
  WHERE AccountNumber = 200
  AND AccountType LIKE '%Daily%';

CREATE TABLE #x(i INT PRIMARY KEY, c NCHAR(1), a NCHAR(1));

WHILE @i <= LEN(@var)
BEGIN
  INSERT #x 
    SELECT SUBSTRING(@var, @i, 1), ASCII(SUBSTRING(@var, @i, 1));

  SET @i = @i + 1;
END

SELECT i,c,a FROM #x ORDER BY i;

You might also consider doing better cleansing of this data before it gets into your database. Cleaning it every time you need to search or display is not the best approach.

How to find a string inside a entire database?

Here are couple more free tools that can be used for this. Both work as SSMS addins.

ApexSQL Search – 100% free - searches both schema and data in tables. Has couple more useful options such as dependency tracking…

SSMS Tools pack – free for all versions except SQL 2012 – doesn’t look as advanced as previous one but has a lot of other cool features.

How do I execute cmd commands through a batch file?

This fixes some issues with Blorgbeard's answer (but is untested):

@echo off
cd /d "c:\Program files\IIS Express"
start "" iisexpress /path:"C:\FormsAdmin.Site" /port:8088 /clr:v2.0
timeout 10
start http://localhost:8088/default.aspx
pause

How do I read a specified line in a text file?

Tried and tested. It's as simple as follows:

string line = File.ReadLines(filePath).ElementAt(actualLineNumber - 1);

As long as you have a text file, this should work. Later, depending upon what data you expect to read, you can cast the string accordingly and use it.

fatal: ambiguous argument 'origin': unknown revision or path not in the working tree

For those experiencing this error on CI/CD, adding the line below worked for me on my GitHub Actions CI/CD workflow right after running pip install pyflakes diff-cover:

git fetch origin master:refs/remotes/origin/master

This is a snippet of the solution from the diff-cover github repo:

Solution: diff-cover matches source files in the coverage XML report with source files in the git diff. For this reason, it's important that the relative paths to the files match. If you are using coverage.py to generate the coverage XML report, then make sure you run diff-cover from the same working directory.

I got the solution on the links below. It is a documented diff-cover error.

https://diff-cover.readthedocs.io/en/latest//README.html https://github.com/Bachmann1234/diff_cover/blob/master/README.rst

Hope this helps :-).

spring autowiring with unique beans: Spring expected single matching bean but found 2

For me it was case of having two beans implementing the same interface. One was a fake ban for the sake of unit test which was conflicting with original bean. If we use

@component("suggestionServicefake")

, it still references with suggestionService. So I removed @component and only used

@Qualifier("suggestionServicefake")

which solved the problem

How to get the clicked link's href with jquery?

Suppose we have three anchor tags like ,

<a  href="ID=1" class="testClick">Test1.</a>
<br />
<a  href="ID=2" class="testClick">Test2.</a>
<br />
<a  href="ID=3" class="testClick">Test3.</a>

now in script

$(".testClick").click(function () {
        var anchorValue= $(this).attr("href");
        alert(anchorValue);
});

use this keyword instead of className (testClick)

SSIS Excel Connection Manager failed to Connect to the Source

I faced the same issue. I think @Rishit answer helped me. This issue is related to 32 bit/ 64 bit version of driver. I was trying to read .xlsx files to SQL Server tables using SSIS

  • My machine was pre-installed with Office 2016 64 bit on Win 10 machine along with MS Access
  • I was able to read excel 97-2003 (.xls) files using ssis, but unable to connect .xlsx files
  • My requirement was to read .xlsx files
  • Installed AccessDatabaseEngine_X64 to read xlsx, that given me the following error:

enter image description here

  • I uninstalled the AccessDatabaseEngine_X64 and installed AccessDatabaseEngine 32 bit, that resolved the issue

Better way to Format Currency Input editText?

Here is my custom CurrencyEditText

import android.content.Context;import android.graphics.Rect;import android.text.Editable;import android.text.InputFilter;import android.text.InputType;import android.text.TextWatcher;
import android.util.AttributeSet;import android.widget.EditText;import java.math.BigDecimal;import java.math.RoundingMode;
import java.text.DecimalFormat;import java.text.DecimalFormatSymbols;
import java.util.Locale;

/**
 * Some note <br/>
 * <li>Always use locale US instead of default to make DecimalFormat work well in all language</li>
 */
public class CurrencyEditText extends android.support.v7.widget.AppCompatEditText {
    private static String prefix = "VND ";
    private static final int MAX_LENGTH = 20;
    private static final int MAX_DECIMAL = 3;
    private CurrencyTextWatcher currencyTextWatcher = new CurrencyTextWatcher(this, prefix);

    public CurrencyEditText(Context context) {
        this(context, null);
    }

    public CurrencyEditText(Context context, AttributeSet attrs) {
        this(context, attrs, android.support.v7.appcompat.R.attr.editTextStyle);
    }

    public CurrencyEditText(Context context, AttributeSet attrs, int defStyleAttr) {
        super(context, attrs, defStyleAttr);
        this.setInputType(InputType.TYPE_CLASS_NUMBER | InputType.TYPE_NUMBER_FLAG_DECIMAL);
        this.setHint(prefix);
        this.setFilters(new InputFilter[] { new InputFilter.LengthFilter(MAX_LENGTH) });
    }

    @Override
    protected void onFocusChanged(boolean focused, int direction, Rect previouslyFocusedRect) {
        super.onFocusChanged(focused, direction, previouslyFocusedRect);
        if (focused) {
            this.addTextChangedListener(currencyTextWatcher);
        } else {
            this.removeTextChangedListener(currencyTextWatcher);
        }
        handleCaseCurrencyEmpty(focused);
    }

    /**
     * When currency empty <br/>
     * + When focus EditText, set the default text = prefix (ex: VND) <br/>
     * + When EditText lose focus, set the default text = "", EditText will display hint (ex:VND)
     */
    private void handleCaseCurrencyEmpty(boolean focused) {
        if (focused) {
            if (getText().toString().isEmpty()) {
                setText(prefix);
            }
        } else {
            if (getText().toString().equals(prefix)) {
                setText("");
            }
        }
    }

    private static class CurrencyTextWatcher implements TextWatcher {
        private final EditText editText;
        private String previousCleanString;
        private String prefix;

        CurrencyTextWatcher(EditText editText, String prefix) {
            this.editText = editText;
            this.prefix = prefix;
        }

        @Override
        public void beforeTextChanged(CharSequence s, int start, int count, int after) {
            // do nothing
        }

        @Override
        public void onTextChanged(CharSequence s, int start, int before, int count) {
            // do nothing
        }

        @Override
        public void afterTextChanged(Editable editable) {
            String str = editable.toString();
            if (str.length() < prefix.length()) {
                editText.setText(prefix);
                editText.setSelection(prefix.length());
                return;
            }
            if (str.equals(prefix)) {
                return;
            }
            // cleanString this the string which not contain prefix and ,
            String cleanString = str.replace(prefix, "").replaceAll("[,]", "");
            // for prevent afterTextChanged recursive call
            if (cleanString.equals(previousCleanString) || cleanString.isEmpty()) {
                return;
            }
            previousCleanString = cleanString;

            String formattedString;
            if (cleanString.contains(".")) {
                formattedString = formatDecimal(cleanString);
            } else {
                formattedString = formatInteger(cleanString);
            }
            editText.removeTextChangedListener(this); // Remove listener
            editText.setText(formattedString);
            handleSelection();
            editText.addTextChangedListener(this); // Add back the listener
        }

        private String formatInteger(String str) {
            BigDecimal parsed = new BigDecimal(str);
            DecimalFormat formatter =
                    new DecimalFormat(prefix + "#,###", new DecimalFormatSymbols(Locale.US));
            return formatter.format(parsed);
        }

        private String formatDecimal(String str) {
            if (str.equals(".")) {
                return prefix + ".";
            }
            BigDecimal parsed = new BigDecimal(str);
            // example pattern VND #,###.00
            DecimalFormat formatter = new DecimalFormat(prefix + "#,###." + getDecimalPattern(str),
                    new DecimalFormatSymbols(Locale.US));
            formatter.setRoundingMode(RoundingMode.DOWN);
            return formatter.format(parsed);
        }

        /**
         * It will return suitable pattern for format decimal
         * For example: 10.2 -> return 0 | 10.23 -> return 00, | 10.235 -> return 000
         */
        private String getDecimalPattern(String str) {
            int decimalCount = str.length() - str.indexOf(".") - 1;
            StringBuilder decimalPattern = new StringBuilder();
            for (int i = 0; i < decimalCount && i < MAX_DECIMAL; i++) {
                decimalPattern.append("0");
            }
            return decimalPattern.toString();
        }

        private void handleSelection() {
            if (editText.getText().length() <= MAX_LENGTH) {
                editText.setSelection(editText.getText().length());
            } else {
                editText.setSelection(MAX_LENGTH);
            }
        }
    }
}

Use it in XML like

 <...CurrencyEditText
        android:layout_width="match_parent"
        android:layout_height="wrap_content"
        />

You should edit 2 constant below for suitable for your project

private static String prefix = "VND ";
private static final int MAX_DECIMAL = 3;

enter image description here

Demo on github

cmd line rename file with date and time

I tried to do the same:

<fileName>.<ext> --> <fileName>_<date>_<time>.<ext> 

I found that :

rename 's/(\w+)(\.\w+)/$1'$(date +"%Y%m%d_%H%M%S)'$2/' *

Expanding tuples into arguments

Take a look at the Python tutorial section 4.7.3 and 4.7.4. It talks about passing tuples as arguments.

I would also consider using named parameters (and passing a dictionary) instead of using a tuple and passing a sequence. I find the use of positional arguments to be a bad practice when the positions are not intuitive or there are multiple parameters.

Drawable image on a canvas

The good way to draw a Drawable on a canvas is not decoding it yourself but leaving it to the system to do so:

Drawable d = getResources().getDrawable(R.drawable.foobar, null);
d.setBounds(left, top, right, bottom);
d.draw(canvas);

This will work with all kinds of drawables, not only bitmaps. And it also means that you can re-use that same drawable again if only the size changes.

How to crop a CvMat in OpenCV?

I understand this question has been answered but perhaps this might be useful to someone...

If you wish to copy the data into a separate cv::Mat object you could use a function similar to this:

void ExtractROI(Mat& inImage, Mat& outImage, Rect roi){
    /* Create the image */
    outImage = Mat(roi.height, roi.width, inImage.type(), Scalar(0));

    /* Populate the image */
    for (int i = roi.y; i < (roi.y+roi.height); i++){
        uchar* inP = inImage.ptr<uchar>(i);
        uchar* outP = outImage.ptr<uchar>(i-roi.y);
        for (int j = roi.x; j < (roi.x+roi.width); j++){
            outP[j-roi.x] = inP[j];
        }
    }
}

It would be important to note that this would only function properly on single channel images.

Java - Search for files in a directory

public class searchingFile 
{
     static String path;//defining(not initializing) these variables outside main 
     static String filename;//so that recursive function can access them
     static int counter=0;//adding static so that can be accessed by static methods 

    public static void main(String[] args) //main methods begins
    {
        Scanner sc=new Scanner(System.in);
        System.out.println("Enter the path : ");
        path=sc.nextLine(); //storing path in path variable
        System.out.println("Enter file name : ");
        filename=sc.nextLine(); //storing filename in filename variable
        searchfile(path);//calling our recursive function and passing path as argument
        System.out.println("Number of locations file found at : "+counter);//Printing occurences

    }

    public static String searchfile(String path)//declaring recursive function having return 
                                                //type and argument both strings

    {
        File file=new File(path);//denoting the path
        File[] filelist=file.listFiles();//storing all the files and directories in array

    for (int i = 0; i < filelist.length; i++) //for loop for accessing all resources
    {
        if(filelist[i].getName().equals(filename))//if loop is true if resource name=filename
        {
            System.out.println("File is present at : "+filelist[i].getAbsolutePath());
            //if loop is true,this will print it's location
            counter++;//counter increments if file found
        }
        if(filelist[i].isDirectory())// if resource is a directory,we want to inside that folder
        {
            path=filelist[i].getAbsolutePath();//this is the path of the subfolder
            searchfile(path);//this path is again passed into the searchfile function 
                             //and this countinues untill we reach a file which has
                             //no sub directories

        }
    }
    return path;// returning path variable as it is the return type and also 
                // because function needs path as argument.

    }   
}

C# equivalent of C++ map<string,double>

This code is all you need:

   static void Main(string[] args) {
        String xml = @"
            <transactions>
                <transaction name=""Fred"" amount=""5,20"" />
                <transaction name=""John"" amount=""10,00"" />
                <transaction name=""Fred"" amount=""3,00"" />
            </transactions>";

        XDocument xmlDocument = XDocument.Parse(xml);

        var query = from x in xmlDocument.Descendants("transaction")
                    group x by x.Attribute("name").Value into g
                    select new { Name = g.Key, Amount = g.Sum(t => Decimal.Parse(t.Attribute("amount").Value)) };

        foreach (var item in query) {
            Console.WriteLine("Name: {0}; Amount: {1:C};", item.Name, item.Amount);
        }
    }

And the content is:

Name: Fred; Amount: R$ 8,20;
Name: John; Amount: R$ 10,00;

That is the way of doing this in C# - in a declarative way!

I hope this helps,

Ricardo Lacerda Castelo Branco

Set session variable in laravel

In Laravel 6.x

// Retrieve a piece of data from the session...
$value = session('key');

// Specifying a default value...
$value = session('key', 'default');

// Store a piece of data in the session...
session(['key' => 'value']);

https://laravel.com/docs/6.x/session

Add a tooltip to a div

Try this. You can do it with only css and I have only added data-title attribute for tooltip.

_x000D_
_x000D_
.tooltip{_x000D_
  position:relative;_x000D_
  display: inline-block;_x000D_
}_x000D_
.tooltip[data-title]:hover:after {_x000D_
  content: attr(data-title);_x000D_
  padding: 4px 8px;_x000D_
  color: #fff;_x000D_
  position: absolute;_x000D_
  left: 0;_x000D_
  top: 110%;_x000D_
  white-space: nowrap;  _x000D_
  border-radius: 5px;  _x000D_
  background:#000;_x000D_
}
_x000D_
<div data-title="My tooltip" class="tooltip">_x000D_
    <label>Name</label>_x000D_
    <input type="text"/>_x000D_
</div>_x000D_
    
_x000D_
_x000D_
_x000D_

How do you use Intent.FLAG_ACTIVITY_CLEAR_TOP to clear the Activity Stack?

i called activity_name.this.finish() after starting new intent and it worked for me.

I tried "FLAG_ACTIVITY_CLEAR_TOP" and "FLAG_ACTIVITY_NEW_TASK"

But it won't work for me... I am not suggesting this solution for use but if setting flag won't work for you than you can try this..But still i recommend don't use it

Create a folder and sub folder in Excel VBA

There are some good answers on here, so I will just add some process improvements. A better way of determining if the folder exists (does not use FileSystemObjects, which not all computers are allowed to use):

Function FolderExists(FolderPath As String) As Boolean
     FolderExists = True
     On Error Resume Next
     ChDir FolderPath
     If Err <> 0 Then FolderExists = False
     On Error GoTo 0
End Function

Likewise,

Function FileExists(FileName As String) As Boolean
     If Dir(FileName) <> "" Then FileExists = True Else FileExists = False
EndFunction

Click event on select option element in chrome

Looking for this on 2018. Click event on option tag, inside a select tag, is not fired on Chrome.

Use change event, and capture the selected option:

$(document).delegate("select", "change", function() {
    //capture the option
    var $target = $("option:selected",$(this));
});

Be aware that $target may be a collection of objects if the select tag is multiple.

center a row using Bootstrap 3

I know this question was specifically targeted at Bootstrap 3, but in case Bootstrap 4 users stumble upon this question, here is how i centered rows in v4:

<div class="container">
  <div class="row justify-content-center">
  ...

More related to this topic can be found on bootstrap site.

Search in all files in a project in Sublime Text 3

Solution:

Use the Search all shortcut: Ctrl+Shift+F, then select the folder in the "Where:" box below. (And for Mac, it's ?+Shift+F).

If the root directory for the project is proj, with subdirectories src and aux and you want to search in all subfolders, use the proj folder. To restrict the search to only the src folder, use proj/src in the "Where: " box.

Embed Youtube video inside an Android app

there is an official YouTube Android Player API wich you can use. This is a bit more complicated but it is working better than other solutions using webclients.

First you must register your app in Googles API Console. This is completely free until your app gets over 25k request a month (or something like that). There are complete anf great tutorials under the link. I hope you can understand them. If not, ask! :)

convert datetime to date format dd/mm/yyyy

You have to pass the CultureInfo to get the result with slash(/)

DateTime.Now.ToString("dd/MM/yyyy", CultureInfo.InvariantCulture)

BigDecimal setScale and round

One important point that is alluded to but not directly addressed is the difference between "precision" and "scale" and how they are used in the two statements. "precision" is the total number of significant digits in a number. "scale" is the number of digits to the right of the decimal point.

The MathContext constructor only accepts precision and RoundingMode as arguments, and therefore scale is never specified in the first statement.

setScale() obviously accepts scale as an argument, as well as RoundingMode, however precision is never specified in the second statement.

If you move the decimal point one place to the right, the difference will become clear:

// 1.
new BigDecimal("35.3456").round(new MathContext(4, RoundingMode.HALF_UP));
//result = 35.35
// 2.
new BigDecimal("35.3456").setScale(4, RoundingMode.HALF_UP);
// result = 35.3456

Error: Module not specified (IntelliJ IDEA)

Faced the same issue. To solve it,

How to add a new line of text to an existing file in Java?

You can use the FileWriter(String fileName, boolean append) constructor if you want to append data to file.

Change your code to this:

output = new BufferedWriter(new FileWriter(my_file_name, true));

From FileWriter javadoc:

Constructs a FileWriter object given a file name. If the second argument is true, then bytes will be written to the end of the file rather than the beginning.

Cannot issue data manipulation statements with executeQuery()

When executing DML statement , you should use executeUpdate/execute rather than executeQuery.

Here is a brief comparison :

executeQueryVSexecuteUpdateVSexecute

Tomcat manager/html is not available?

I had the situatuion when tomcat manager did not start. I had this exception in my logs/manager.DDD-MM-YY.log:

org.apache.catalina.core.StandardContext filterStart
SEVERE: Exception starting filter CSRF
java.lang.ClassNotFoundException: org.apache.catalina.filters.CsrfPreventionFilter
        at java.net.URLClassLoader$1.run(URLClassLoader.java:202)
        ...

This exception was raised because I used a version of tomcat that hadn't CSRF prevention filter. Tomcat 6.0.24 doesn't have the CSRF prevention filter in it. The first version that has it is the 6.0.30 version (at least. according to the changelog). As a result, Tomcat Manager was uncompatible with version of Tomcat that I used. I've digged description of this issue here: http://blog.techstacks.com/.m/2009/05/tomcat-management-setting-up-tomcat/comments/

Steps to fix it:

  1. Check version of tomcat installed by running "sh version.sh" from your tomcat/bin directory
  2. Download corresponding version of tomcat
  3. Stop tomcat
  4. Remove your webapps/manager directory and copy manager application from distributive that you've downloaded.
  5. Start tomcat

Now you should be able to access tomcat manager.

Interfaces with static fields in java for sharing 'constants'

This came from a time before Java 1.5 exists and bring enums to us. Prior to that, there was no good way to define a set of constants or constrained values.

This is still used, most of the time either for backward compatibility or due to the amount of refactoring needed to get rid off, in a lot of project.

Warning: comparison with string literals results in unspecified behaviour

This an old question, but I have had to explain it to someone recently and I thought recording the answer here would be helpful at least in understanding how C works.

String literals like

"a"

or

"This is a string"

are put in the text or data segments of your program.

A string in C is actually a pointer to a char, and the string is understood to be the subsequent chars in memory up until a NUL char is encountered. That is, C doesn't really know about strings.

So if I have

char *s1 = "This is a string";

then s1 is a pointer to the first byte of the string.

Now, if I have

char *s2 = "This is a string";

this is also a pointer to the same first byte of that string in the text or data segment of the program.

But if I have

char *s3 = malloc( 17 );
strcpy(s3, "This is a string");

then s3 is a pointer to another place in memory into which I copy all the bytes of the other strings.

Illustrative examples:

Although, as your compiler rightly points out, you shouldn't do this, the following will evaluate to true:

s1 == s2 // True: we are comparing two pointers that contain the same address

but the following will evaluate to false

s1 == s3 // False: Comparing two pointers that don't hold the same address.

And although it might be tempting to have something like this:

struct Vehicle{
    char *type;
    // other stuff
}

if( type == "Car" )
   //blah1
else if( type == "Motorcycle )
   //blah2

You shouldn't do it because it's not something that is guarantied to work. Even if you know that type will always be set using a string literal.

I have tested it and it works. If I do

A.type = "Car";

then blah1 gets executed and similarly for "Motorcycle". And you'd be able to do things like

if( A.type == B.type )

but this is just terrible. I'm writing about it because I think it's interesting to know why it works, and it helps understand why you shouldn't do it.

Solutions:

In your case, what you want to do is use strcmp(a,b) == 0 to replace a == b

In the case of my example, you should use an enum.

enum type {CAR = 0, MOTORCYCLE = 1}

The preceding thing with string was useful because you could print the type, so you might have an array like this

char *types[] = {"Car", "Motorcycle"};

And now that I think about it, this is error prone since one must be careful to maintain the same order in the types array.

Therefore it might be better to do

char *getTypeString(int type)
{
    switch(type)
    case CAR: return "Car";
    case MOTORCYCLE: return "Motorcycle"
    default: return NULL;
}

Retrofit 2 - Dynamic URL

You can use the encoded flag on the @Path annotation:

public interface APIService {
  @GET("{fullUrl}")
  Call<Users> getUsers(@Path(value = "fullUrl", encoded = true) String fullUrl);
}
  • This will prevent the replacement of / with %2F.
  • It will not save you from ? being replaced by %3F, however, so you still can't pass in dynamic query strings.

Performing a query on a result from another query?

I don't know if you even need to wrap it. Won't this work?

SELECT COUNT(*), SUM(DATEDIFF(now(),availables.updated_at))
FROM availables
INNER JOIN rooms    ON availables.room_id=rooms.id
WHERE availables.bookdate BETWEEN '2009-06-25' 
  AND date_add('2009-06-25', INTERVAL 4 DAY)
  AND rooms.hostel_id = 5094
GROUP BY availables.bookdate);

If your goal is to return both result sets then you'll need to store it some place temporarily.

Git push won't do anything (everything up-to-date)

This happened to me when I ^C in the middle of a git push to GitHub. GitHub did not show that the changes had been made, however.

To fix it, I made a change to my working tree, committed, and then pushed again. It worked perfectly fine.

C# : "A first chance exception of type 'System.InvalidOperationException'"

The problem here is that your timer starts a thread and when it runs the callback function, the callback function ( updatelistview) is accessing controls on UI thread so this can not be done becuase of this

JQuery - Set Attribute value

Seriously, just don't use jQuery for this. disabled is a boolean property of form elements that works perfectly in every major browser since 1997, and there is no possible way it could be simpler or more intuitive to change whether or not a form element is disabled.

The simplest way of getting a reference to the checkbox would be to give it an id. Here's my suggested HTML:

<input type="hidden" name="chk0" value="">
<input type="checkbox" name="chk0" id="chk0_checkbox" value="true" disabled>

And the line of JavaScript to make the check box enabled:

document.getElementById("chk0_checkbox").disabled = false;

If you prefer, you can instead use jQuery to get hold of the checkbox:

$("#chk0_checkbox")[0].disabled = false;

How to ignore files/directories in TFS for avoiding them to go to central source repository?

For VS2015 and VS2017

Works with TFS (on-prem) or VSO (Visual Studio Online - the Azure-hosted offering)

The NuGet documentation provides instructions on how to accomplish this and I just followed them successfully for Visual Studio 2015 & Visual Studio 2017 against VSTS (Azure-hosted TFS). Everything is fully updated as of Nov 2016 Aug 2018.

I recommend you follow NuGet's instructions but just to recap what I did:

  1. Make sure your packages folder is not committed to TFS. If it is, get it out of there.
  2. Everything else we create below goes into the same folder that your .sln file exists in unless otherwise specified (NuGet's instructions aren't completely clear on this).
  3. Create a .nuget folder. You can use Windows Explorer to name it .nuget. for it to successfully save as .nuget (it automatically removes the last period) but directly trying to name it .nuget may not work (you may get an error or it may change the name, depending on your version of Windows). Or name the directory nuget, and open the parent directory in command line prompt. type. ren nuget .nuget
  4. Inside of that folder, create a NuGet.config file and add the following contents and save it:

NuGet.config:

<?xml version="1.0" encoding="utf-8"?>
<configuration>
    <solution>
        <add key="disableSourceControlIntegration" value="true" />
    </solution>
</configuration>
  1. Go back in your .sln's folder and create a new text file and name it .tfignore (if using Windows Explorer, use the same trick as above and name it .tfignore.)
  2. Put the following content into that file:

.tfignore:

# Ignore the NuGet packages folder in the root of the repository.
# If needed, prefix 'packages' with additional folder names if it's 
# not in the same folder as .tfignore.
packages

# include package target files which may be required for msbuild,
# again prefixing the folder name as needed.
!packages/*.targets
  1. Save all of this, commit it to TFS, then close & re-open Visual Studio and the Team Explorer should no longer identify the packages folder as a pending check-in.
  2. Copy/pasted via Windows Explorer the .tfignore file and .nuget folder to all of my various solutions and committed them and I no longer have the packages folder trying to sneak into my source control repo!

Further Customization

While not mine, I have found this .tfignore template by sirkirby to be handy. The example in my answer covers the Nuget packages folder but this template includes some other things as well as provides additional examples that can be useful if you wish to customize this further.

AngularJS UI Router - change url without reloading state

Ok, solved :) Angular UI Router has this new method, $urlRouterProvider.deferIntercept() https://github.com/angular-ui/ui-router/issues/64

basically it comes down to this:

angular.module('myApp', [ui.router])
  .config(['$urlRouterProvider', function ($urlRouterProvider) {
    $urlRouterProvider.deferIntercept();
  }])
  // then define the interception
  .run(['$rootScope', '$urlRouter', '$location', '$state', function ($rootScope, $urlRouter, $location, $state) {
    $rootScope.$on('$locationChangeSuccess', function(e, newUrl, oldUrl) {
      // Prevent $urlRouter's default handler from firing
      e.preventDefault();

      /** 
       * provide conditions on when to 
       * sync change in $location.path() with state reload.
       * I use $location and $state as examples, but
       * You can do any logic
       * before syncing OR stop syncing all together.
       */

      if ($state.current.name !== 'main.exampleState' || newUrl === 'http://some.url' || oldUrl !=='https://another.url') {
        // your stuff
        $urlRouter.sync();
      } else {
        // don't sync
      }
    });
    // Configures $urlRouter's listener *after* your custom listener
    $urlRouter.listen();
  }]);

I think this method is currently only included in the master version of angular ui router, the one with optional parameters (which are nice too, btw). It needs to be cloned and built from source with

grunt build

The docs are accessible from the source as well, through

grunt ngdocs

(they get built into the /site directory) // more info in README.MD

There seems to be another way to do this, by dynamic parameters (which I haven't used). Many credits to nateabele.


As a sidenote, here are optional parameters in Angular UI Router's $stateProvider, which I used in combination with the above:

angular.module('myApp').config(['$stateProvider', function ($stateProvider) {    

  $stateProvider
    .state('main.doorsList', {
      url: 'doors',
      controller: DoorsListCtrl,
      resolve: DoorsListCtrl.resolve,
      templateUrl: '/modules/doors/doors-list.html'
    })
    .state('main.doorsSingle', {
      url: 'doors/:doorsSingle/:doorsDetail',
      params: {
        // as of today, it was unclear how to define a required parameter (more below)
        doorsSingle: {value: null},
        doorsDetail: {value: null}
      },
      controller: DoorsSingleCtrl,
      resolve: DoorsSingleCtrl.resolve,
      templateUrl: '/modules/doors/doors-single.html'
    });

}]);

what that does is it allows to resolve a state, even if one of the params is missing. SEO is one purpose, readability another.

In the example above, I wanted doorsSingle to be a required parameter. It is not clear how to define those. It works ok with multiple optional parameters though, so not really a problem. The discussion is here https://github.com/angular-ui/ui-router/pull/1032#issuecomment-49196090

What is the difference between float and double?

When using floating point numbers you cannot trust that your local tests will be exactly the same as the tests that are done on the server side. The environment and the compiler are probably different on you local system and where the final tests are run. I have seen this problem many times before in some TopCoder competitions especially if you try to compare two floating point numbers.

How to send/receive SOAP request and response using C#?

The urls are different.

  • http://localhost/AccountSvc/DataInquiry.asmx

vs.

  • /acctinqsvc/portfolioinquiry.asmx

Resolve this issue first, as if the web server cannot resolve the URL you are attempting to POST to, you won't even begin to process the actions described by your request.

You should only need to create the WebRequest to the ASMX root URL, ie: http://localhost/AccountSvc/DataInquiry.asmx, and specify the desired method/operation in the SOAPAction header.

The SOAPAction header values are different.

  • http://localhost/AccountSvc/DataInquiry.asmx/ + methodName

vs.

  • http://tempuri.org/GetMyName

You should be able to determine the correct SOAPAction by going to the correct ASMX URL and appending ?wsdl

There should be a <soap:operation> tag underneath the <wsdl:operation> tag that matches the operation you are attempting to execute, which appears to be GetMyName.

There is no XML declaration in the request body that includes your SOAP XML.

You specify text/xml in the ContentType of your HttpRequest and no charset. Perhaps these default to us-ascii, but there's no telling if you aren't specifying them!

The SoapUI created XML includes an XML declaration that specifies an encoding of utf-8, which also matches the Content-Type provided to the HTTP request which is: text/xml; charset=utf-8

Hope that helps!

ICommand MVVM implementation

This is almost identical to how Karl Shifflet demonstrated a RelayCommand, where Execute fires a predetermined Action<T>. A top-notch solution, if you ask me.

public class RelayCommand : ICommand
{
    private readonly Predicate<object> _canExecute;
    private readonly Action<object> _execute;

    public RelayCommand(Predicate<object> canExecute, Action<object> execute)
    {
        _canExecute = canExecute;
        _execute = execute;
    }

    public event EventHandler CanExecuteChanged
    {
        add => CommandManager.RequerySuggested += value;
        remove => CommandManager.RequerySuggested -= value;
    }

    public bool CanExecute(object parameter)
    {
        return _canExecute(parameter);
    }

    public void Execute(object parameter)
    {
        _execute(parameter);
    }
}

This could then be used as...

public class MyViewModel
{
    private ICommand _doSomething;
    public ICommand DoSomethingCommand
    {
        get
        {
            if (_doSomething == null)
            {
                _doSomething = new RelayCommand(
                    p => this.CanDoSomething,
                    p => this.DoSomeImportantMethod());
            }
            return _doSomething;
        }
    }
}

Read more:
Josh Smith (introducer of RelayCommand): Patterns - WPF Apps With The MVVM Design Pattern

DateTime "null" value

I always set the time to DateTime.MinValue. This way I do not get any NullErrorException and I can compare it to a date that I know isn't set.

jQuery .attr("disabled", "disabled") not working in Chrome

Here:
http://jsbin.com/urize4/edit

Live Preview
http://jsbin.com/urize4/

You should use "readonly" instead like:

$("input[type='text']").attr("readonly", "true");

Using StringWriter for XML Serialization

<TL;DR> The problem is rather simple, actually: you are not matching the declared encoding (in the XML declaration) with the datatype of the input parameter. If you manually added <?xml version="1.0" encoding="utf-8"?><test/> to the string, then declaring the SqlParameter to be of type SqlDbType.Xml or SqlDbType.NVarChar would give you the "unable to switch the encoding" error. Then, when inserting manually via T-SQL, since you switched the declared encoding to be utf-16, you were clearly inserting a VARCHAR string (not prefixed with an upper-case "N", hence an 8-bit encoding, such as UTF-8) and not an NVARCHAR string (prefixed with an upper-case "N", hence the 16-bit UTF-16 LE encoding).

The fix should have been as simple as:

  1. In the first case, when adding the declaration stating encoding="utf-8": simply don't add the XML declaration.
  2. In the second case, when adding the declaration stating encoding="utf-16": either
    1. simply don't add the XML declaration, OR
    2. simply add an "N" to the input parameter type: SqlDbType.NVarChar instead of SqlDbType.VarChar :-) (or possibly even switch to using SqlDbType.Xml)

(Detailed response is below)


All of the answers here are over-complicated and unnecessary (regardless of the 121 and 184 up-votes for Christian's and Jon's answers, respectively). They might provide working code, but none of them actually answer the question. The issue is that nobody truly understood the question, which ultimately is about how the XML datatype in SQL Server works. Nothing against those two clearly intelligent people, but this question has little to nothing to do with serializing to XML. Saving XML data into SQL Server is much easier than what is being implied here.

It doesn't really matter how the XML is produced as long as you follow the rules of how to create XML data in SQL Server. I have a more thorough explanation (including working example code to illustrate the points outlined below) in an answer on this question: How to solve “unable to switch the encoding” error when inserting XML into SQL Server, but the basics are:

  1. The XML declaration is optional
  2. The XML datatype stores strings always as UCS-2 / UTF-16 LE
  3. If your XML is UCS-2 / UTF-16 LE, then you:
    1. pass in the data as either NVARCHAR(MAX) or XML / SqlDbType.NVarChar (maxsize = -1) or SqlDbType.Xml, or if using a string literal then it must be prefixed with an upper-case "N".
    2. if specifying the XML declaration, it must be either "UCS-2" or "UTF-16" (no real difference here)
  4. If your XML is 8-bit encoded (e.g. "UTF-8" / "iso-8859-1" / "Windows-1252"), then you:
    1. need to specify the XML declaration IF the encoding is different than the code page specified by the default Collation of the database
    2. you must pass in the data as VARCHAR(MAX) / SqlDbType.VarChar (maxsize = -1), or if using a string literal then it must not be prefixed with an upper-case "N".
    3. Whatever 8-bit encoding is used, the "encoding" noted in the XML declaration must match the actual encoding of the bytes.
    4. The 8-bit encoding will be converted into UTF-16 LE by the XML datatype

With the points outlined above in mind, and given that strings in .NET are always UTF-16 LE / UCS-2 LE (there is no difference between those in terms of encoding), we can answer your questions:

Is there a reason why I shouldn't use StringWriter to serialize an Object when I need it as a string afterwards?

No, your StringWriter code appears to be just fine (at least I see no issues in my limited testing using the 2nd code block from the question).

Wouldn't setting the encoding to UTF-16 (in the xml tag) work then?

It isn't necessary to provide the XML declaration. When it is missing, the encoding is assumed to be UTF-16 LE if you pass the string into SQL Server as NVARCHAR (i.e. SqlDbType.NVarChar) or XML (i.e. SqlDbType.Xml). The encoding is assumed to be the default 8-bit Code Page if passing in as VARCHAR (i.e. SqlDbType.VarChar). If you have any non-standard-ASCII characters (i.e. values 128 and above) and are passing in as VARCHAR, then you will likely see "?" for BMP characters and "??" for Supplementary Characters as SQL Server will convert the UTF-16 string from .NET into an 8-bit string of the current Database's Code Page before converting it back into UTF-16 / UCS-2. But you shouldn't get any errors.

On the other hand, if you do specify the XML declaration, then you must pass into SQL Server using the matching 8-bit or 16-bit datatype. So if you have a declaration stating that the encoding is either UCS-2 or UTF-16, then you must pass in as SqlDbType.NVarChar or SqlDbType.Xml. Or, if you have a declaration stating that the encoding is one of the 8-bit options (i.e. UTF-8, Windows-1252, iso-8859-1, etc), then you must pass in as SqlDbType.VarChar. Failure to match the declared encoding with the proper 8 or 16 -bit SQL Server datatype will result in the "unable to switch the encoding" error that you were getting.

For example, using your StringWriter-based serialization code, I simply printed the resulting string of the XML and used it in SSMS. As you can see below, the XML declaration is included (because StringWriter does not have an option to OmitXmlDeclaration like XmlWriter does), which poses no problem so long as you pass the string in as the correct SQL Server datatype:

-- Upper-case "N" prefix == NVARCHAR, hence no error:
DECLARE @Xml XML = N'<?xml version="1.0" encoding="utf-16"?>
<string>Test ?</string>';
SELECT @Xml;
-- <string>Test ?</string>

As you can see, it even handles characters beyond standard ASCII, given that ? is BMP Code Point U+1234, and is Supplementary Character Code Point U+1F638. However, the following:

-- No upper-case "N" prefix on the string literal, hence VARCHAR:
DECLARE @Xml XML = '<?xml version="1.0" encoding="utf-16"?>
<string>Test ?</string>';

results in the following error:

Msg 9402, Level 16, State 1, Line XXXXX
XML parsing: line 1, character 39, unable to switch the encoding

Ergo, all of that explanation aside, the full solution to your original question is:

You were clearly passing the string in as SqlDbType.VarChar. Switch to SqlDbType.NVarChar and it will work without needing to go through the extra step of removing the XML declaration. This is preferred over keeping SqlDbType.VarChar and removing the XML declaration because this solution will prevent data loss when the XML includes non-standard-ASCII characters. For example:

-- No upper-case "N" prefix on the string literal == VARCHAR, and no XML declaration:
DECLARE @Xml2 XML = '<string>Test ?</string>';
SELECT @Xml2;
-- <string>Test ???</string>

As you can see, there is no error this time, but now there is data-loss 🙀.

"ssl module in Python is not available" when installing package with pip3

I tried A LOT of ways to solve this problem and none solved. I'm currently on Windows 10.

The only thing that worked was:

  • Uninstall Anaconda
  • Uninstall Python (i was using version 3.7.3)
  • Install Python again (remember to check the option to automatically add to PATH)

Then I've downloaded all the libs I needed using PIP... and worked!

Don't know why, or if the problem was somehow related to Anaconda.

sqlite3.OperationalError: unable to open database file

Run into the error on Windows, added assert os.path.exists, double checked the path, run the script as administrator, nothing helped.

Turns out if you add your folders to the Windows Defender's Ransomware Protection, you can no longer use other programs to write there unless you add these programs to the Controlled Folder Access' whitelist.

Solution - check if your folder has been added to the Windows Defender's Ransomware Protection and remove it for faster fix.

Finding Variable Type in JavaScript

To be a little more ECMAScript-5.1-precise than the other answers (some might say pedantic):

In JavaScript, variables (and properties) don't have types: values do. Further, there are only 6 types of values: Undefined, Null, Boolean, String, Number, and Object. (Technically, there are also 7 "specification types", but you can't store values of those types as properties of objects or values of variables--they are only used within the spec itself, to define how the language works. The values you can explicitly manipulate are of only the 6 types I listed.)

The spec uses the notation "Type(x)" when it wants to talk about "the type of x". This is only a notation used within the spec: it is not a feature of the language.

As other answers make clear, in practice you may well want to know more than the type of a value--particularly when the type is Object. Regardless, and for completeness, here is a simple JavaScript implementation of Type(x) as it is used in the spec:

function Type(x) { 
    if (x === null) {
        return 'Null';
    }

    switch (typeof x) {
    case 'undefined': return 'Undefined';
    case 'boolean'  : return 'Boolean';
    case 'number'   : return 'Number';
    case 'string'   : return 'String';
    default         : return 'Object';
    }
}

Proper use of the IDisposable interface

I won't repeat the usual stuff about Using or freeing un-managed resources, that has all been covered. But I would like to point out what seems a common misconception.
Given the following code

Public Class LargeStuff
  Implements IDisposable
  Private _Large as string()

  'Some strange code that means _Large now contains several million long strings.

  Public Sub Dispose() Implements IDisposable.Dispose
    _Large=Nothing
  End Sub

I realise that the Disposable implementation does not follow current guidelines, but hopefully you all get the idea.
Now, when Dispose is called, how much memory gets freed?

Answer: None.
Calling Dispose can release unmanaged resources, it CANNOT reclaim managed memory, only the GC can do that. Thats not to say that the above isn't a good idea, following the above pattern is still a good idea in fact. Once Dispose has been run, there is nothing stopping the GC re-claiming the memory that was being used by _Large, even though the instance of LargeStuff may still be in scope. The strings in _Large may also be in gen 0 but the instance of LargeStuff might be gen 2, so again, memory would be re-claimed sooner.
There is no point in adding a finaliser to call the Dispose method shown above though. That will just DELAY the re-claiming of memory to allow the finaliser to run.

AngularJS Error: Cross origin requests are only supported for protocol schemes: http, data, chrome-extension, https

RootCause:

File protocol does not support cross origin request for Chrome

Solution 1:

use http protocol instead of file, meaning: set up a http server, such as apache, or nodejs+http-server

Sotution 2:

Add --allow-file-access-from-files after Chrome`s shortcut target, and open new browse instance using this shortcut

enter image description here

Solution 3:

use Firefox instead

In android how to set navigation drawer header image and name programmatically in class file?

First you need to access the navigation drawer in your MainActivity(or the calling activity) like this:

    NavigationView navigationView = (NavigationView) findViewById(R.id.nav_view);

Then you need to remove the header layout from the activity_main.xml because the layout will be inflated programatically in the MainActivity. Your activity_main.xml should look like this:

<?xml version="1.0" encoding="utf-8"?>
<android.support.v4.widget.DrawerLayout            

xmlns:android="http://schemas.android.com/apk/res/android"
xmlns:app="http://schemas.android.com/apk/res-auto"
xmlns:tools="http://schemas.android.com/tools"
android:id="@+id/drawer_layout"
android:layout_width="match_parent"
android:layout_height="match_parent"
android:fitsSystemWindows="true"
tools:openDrawer="start">

<include
    layout="@layout/app_bar_main"
    android:layout_width="match_parent"
    android:layout_height="match_parent" />

<android.support.design.widget.NavigationView
    android:id="@+id/nav_view"
    android:layout_width="wrap_content"
    android:layout_height="match_parent"
    android:layout_gravity="start"
    android:fitsSystemWindows="true"
    app:menu="@menu/activity_main_drawer" />

    </android.support.v4.widget.DrawerLayout>

Then in your MainActivity, we inflate the nav_header_main layout and get access to its views, in this case the ImageView and TextView

//inflate header layout
View navView =  navigationView.inflateHeaderView(R.layout.nav_header_main);
//reference to views
ImageView imgvw = (ImageView)navView.findViewById(R.id.imageView);
TextView tv = (TextView)navView.findViewById(R.id.textview);
//set views
imgvw.setImageResource(R.drawable.your_image);
tv.setText("new text");

navigationView.setNavigationItemSelectedListener(this);

You can read more here

COUNT(*) vs. COUNT(1) vs. COUNT(pk): which is better?

Bottom Line

Use either COUNT(field) or COUNT(*), and stick with it consistently, and if your database allows COUNT(tableHere) or COUNT(tableHere.*), use that.

In short, don't use COUNT(1) for anything. It's a one-trick pony, which rarely does what you want, and in those rare cases is equivalent to count(*)

Use count(*) for counting

Use * for all your queries that need to count everything, even for joins, use *

SELECT boss.boss_id, COUNT(subordinate.*)
FROM boss
LEFT JOIN subordinate on subordinate.boss_id = boss.boss_id
GROUP BY boss.id

But don't use COUNT(*) for LEFT joins, as that will return 1 even if the subordinate table doesn't match anything from parent table

SELECT boss.boss_id, COUNT(*)
FROM boss
LEFT JOIN subordinate on subordinate.boss_id = boss.boss_id
GROUP BY boss.id

Don't be fooled by those advising that when using * in COUNT, it fetches entire row from your table, saying that * is slow. The * on SELECT COUNT(*) and SELECT * has no bearing to each other, they are entirely different thing, they just share a common token, i.e. *.

An alternate syntax

In fact, if it is not permitted to name a field as same as its table name, RDBMS language designer could give COUNT(tableNameHere) the same semantics as COUNT(*). Example:

For counting rows we could have this:

SELECT COUNT(emp) FROM emp

And they could make it simpler:

SELECT COUNT() FROM emp

And for LEFT JOINs, we could have this:

SELECT boss.boss_id, COUNT(subordinate)
FROM boss
LEFT JOIN subordinate on subordinate.boss_id = boss.boss_id
GROUP BY boss.id

But they cannot do that (COUNT(tableNameHere)) since SQL standard permits naming a field with the same name as its table name:

CREATE TABLE fruit -- ORM-friendly name
(
fruit_id int NOT NULL,
fruit varchar(50), /* same name as table name, 
                and let's say, someone forgot to put NOT NULL */
shape varchar(50) NOT NULL,
color varchar(50) NOT NULL
)

Counting with null

And also, it is not a good practice to make a field nullable if its name matches the table name. Say you have values 'Banana', 'Apple', NULL, 'Pears' on fruit field. This will not count all rows, it will only yield 3, not 4

SELECT count(fruit) FROM fruit

Though some RDBMS do that sort of principle (for counting the table's rows, it accepts table name as COUNT's parameter), this will work in Postgresql (if there is no subordinate field in any of the two tables below, i.e. as long as there is no name conflict between field name and table name):

SELECT boss.boss_id, COUNT(subordinate)
FROM boss
LEFT JOIN subordinate on subordinate.boss_id = boss.boss_id
GROUP BY boss.id

But that could cause confusion later if we will add a subordinate field in the table, as it will count the field(which could be nullable), not the table rows.

So to be on the safe side, use:

SELECT boss.boss_id, COUNT(subordinate.*)
FROM boss
LEFT JOIN subordinate on subordinate.boss_id = boss.boss_id
GROUP BY boss.id

count(1): The one-trick pony

In particular to COUNT(1), it is a one-trick pony, it works well only on one table query:

SELECT COUNT(1) FROM tbl

But when you use joins, that trick won't work on multi-table queries without its semantics being confused, and in particular you cannot write:

-- count the subordinates that belongs to boss
SELECT boss.boss_id, COUNT(subordinate.1)
FROM boss
LEFT JOIN subordinate on subordinate.boss_id = boss.boss_id
GROUP BY boss.id

So what's the meaning of COUNT(1) here?

SELECT boss.boss_id, COUNT(1)
FROM boss
LEFT JOIN subordinate on subordinate.boss_id = boss.boss_id
GROUP BY boss.id

Is it this...?

-- counting all the subordinates only
SELECT boss.boss_id, COUNT(subordinate.boss_id)
FROM boss
LEFT JOIN subordinate on subordinate.boss_id = boss.boss_id
GROUP BY boss.id

Or this...?

-- or is that COUNT(1) will also count 1 for boss regardless if boss has a subordinate
SELECT boss.boss_id, COUNT(*)
FROM boss
LEFT JOIN subordinate on subordinate.boss_id = boss.boss_id
GROUP BY boss.id

By careful thought, you can infer that COUNT(1) is the same as COUNT(*), regardless of type of join. But for LEFT JOINs result, we cannot mold COUNT(1) to work as: COUNT(subordinate.boss_id), COUNT(subordinate.*)

So just use either of the following:

-- count the subordinates that belongs to boss
SELECT boss.boss_id, COUNT(subordinate.boss_id)
FROM boss
LEFT JOIN subordinate on subordinate.boss_id = boss.boss_id
GROUP BY boss.id

Works on Postgresql, it's clear that you want to count the cardinality of the set

-- count the subordinates that belongs to boss
SELECT boss.boss_id, COUNT(subordinate.*)
FROM boss
LEFT JOIN subordinate on subordinate.boss_id = boss.boss_id
GROUP BY boss.id

Another way to count the cardinality of the set, very English-like (just don't make a column with a name same as its table name) : http://www.sqlfiddle.com/#!1/98515/7

select boss.boss_name, count(subordinate)
from boss
left join subordinate on subordinate.boss_code = boss.boss_code
group by boss.boss_name

You cannot do this: http://www.sqlfiddle.com/#!1/98515/8

select boss.boss_name, count(subordinate.1)
from boss
left join subordinate on subordinate.boss_code = boss.boss_code
group by boss.boss_name

You can do this, but this produces wrong result: http://www.sqlfiddle.com/#!1/98515/9

select boss.boss_name, count(1)
from boss
left join subordinate on subordinate.boss_code = boss.boss_code
group by boss.boss_name

How do I center this form in css?

Another solution (without a wrapper) would be to set the form to display: table, which would make it act like a table so it would have the width of its largest child, and then apply margin: 0 auto to center it.

form {
    display: table;
    margin: 0 auto;
}

Credit goes to: https://stackoverflow.com/a/49378738/7841955

AngularJS multiple filter with custom filter function

Try this:

<tr ng-repeat="player in players | filter:{id: player_id, name:player_name} | filter:ageFilter">

$scope.ageFilter = function (player) {
    return (player.age > $scope.min_age && player.age < $scope.max_age);
}

ASP.NET MVC How to pass JSON object from View to Controller as Parameter

A different take with a simple jQuery plugin

Even though answers to this question are long overdue, but I'm still posting a nice solution that I came with some time ago and makes it really simple to send complex JSON to Asp.net MVC controller actions so they are model bound to whatever strong type parameters.

This plugin supports dates just as well, so they get converted to their DateTime counterpart without a problem.

You can find all the details in my blog post where I examine the problem and provide code necessary to accomplish this.

All you have to do is to use this plugin on the client side. An Ajax request would look like this:

$.ajax({
    type: "POST",
    url: "SomeURL",
    data: $.toDictionary(yourComplexJSONobject),
    success: function() { ... },
    error: function() { ... }
});

But this is just part of the whole problem. Now we are able to post complex JSON back to server, but since it will be model bound to a complex type that may have validation attributes on properties things may fail at that point. I've got a solution for it as well. My solution takes advantage of jQuery Ajax functionality where results can be successful or erroneous (just as shown in the upper code). So when validation would fail, error function would get called as it's supposed to be.

How can I check if mysql is installed on ubuntu?

You can use tool dpkg for managing packages in Debian operating system.

Example

dpkg --get-selections | grep mysql if it's listed as installed, you got it. Else you need to get it.

How to remove commits from a pull request

So do the following ,

Lets say your branch name is my_branch and this has the extra commits.

  1. git checkout -b my_branch_with_extra_commits (Keeping this branch saved under a different name)
  2. gitk (Opens git console)
  3. Look for the commit you want to keep. Copy the SHA of that commit to a notepad.
  4. git checkout my_branch
  5. gitk (This will open the git console )
  6. Right click on the commit you want to revert to (State before your changes) and click on "reset branch to here"
  7. Do a git pull --rebase origin branch_name_to _merge_to
  8. git cherry-pick <SHA you copied in step 3. >

Now look at the local branch commit history and make sure everything looks good.

What is the difference between Left, Right, Outer and Inner Joins?

There are only 4 kinds:

  1. Inner join: The most common type. An output row is produced for every pair of input rows that match on the join conditions.
  2. Left outer join: The same as an inner join, except that if there is any row for which no matching row in the table on the right can be found, a row is output containing the values from the table on the left, with NULL for each value in the table on the right. This means that every row from the table on the left will appear at least once in the output.
  3. Right outer join: The same as a left outer join, except with the roles of the tables reversed.
  4. Full outer join: A combination of left and right outer joins. Every row from both tables will appear in the output at least once.

A "cross join" or "cartesian join" is simply an inner join for which no join conditions have been specified, resulting in all pairs of rows being output.

Thanks to RusselH for pointing out FULL joins, which I'd omitted.

How to auto import the necessary classes in Android Studio with shortcut?

You can also use Eclipse's keyboard shortcuts: just go on preferences > keymap and choose Eclipse from the drop-down menu. And all your Eclipse shortcuts will be used in here.

How to make HTML Text unselectable

I altered the jQuery plugin posted above so it would work on live elements.

(function ($) {
$.fn.disableSelection = function () {
    return this.each(function () {
        if (typeof this.onselectstart != 'undefined') {
            this.onselectstart = function() { return false; };
        } else if (typeof this.style.MozUserSelect != 'undefined') {
            this.style.MozUserSelect = 'none';
        } else {
            this.onmousedown = function() { return false; };
        }
    });
};
})(jQuery);

Then you could so something like:

$(document).ready(function() {
    $('label').disableSelection();

    // Or to make everything unselectable
    $('*').disableSelection();
});

What is the best way to declare global variable in Vue.js?

In vue cli-3 You can define the variable in main.js like

window.basurl="http://localhost:8000/";

And you can also access this variable in any component by using the the window.basurl

What's the difference between Apache's Mesos and Google's Kubernetes

I like this short video here mesos learning material

with bare metal clusters, you would need to spawn stacks like HDFS, SPARK, MR etc... so if you launch tasks related to these using only bare metal cluster management, there will be a lot cold starting time.

with mesos, you can install these services on top of the bare metals and you can avoid the bring up time of those base services. This is something mesos does well. and can be utilised by kubernetes building on top of it.

findViewByID returns null

I've tried all of the above nothing was working.. so I had to make my ImageView static public static ImageView texture; and then texture = (ImageView) findViewById(R.id.texture_back); , I don't think it's a good approach though but this really worked for my case :)

Simple division in Java - is this a bug or a feature?

You're using integer division.

Try 7.0/10 instead.

how to inherit Constructor from super class to sub class

Superclass constructor CAN'T be inherited in extended class. Although it can be invoked in extended class constructor's with super() as the first statement.

Show and hide divs at a specific time interval using jQuery

Heres a another take on this problem, using recursion and without using mutable variables. Also, im not using setInterval so theres no cleanup that has to be done.

Having this HTML

<section id="testimonials">
    <h2>My testimonial spinner</h2>
    <div class="testimonial">
      <p>First content</p>
    </div>
    <div class="testimonial">
      <p>Second content</p>
    </div>
    <div class="testimonial">
       <p>Third content</p>
    </div>
</section>

Using ES2016

Here you call the function recursively and update the arguments.

  const testimonials = $('#testimonials')
      .children()
      .filter('div.testimonial');

  const showTestimonial = index => {

    testimonials.hide();
    $(testimonials[index]).fadeIn();

    return index === testimonials.length
      ? showTestimonial(0)
      : setTimeout(() => { showTestimonial(index + 1); }, 10000);
   }

showTestimonial(0); // id of the first element you want to show.

How can I Remove .DS_Store files from a Git repository?

When initializing your repository, skip the git command that contains

-u

and it shouldn't be an issue.

Asking the user for input until they give a valid response

The simplest way to accomplish this is to put the input method in a while loop. Use continue when you get bad input, and break out of the loop when you're satisfied.

When Your Input Might Raise an Exception

Use try and except to detect when the user enters data that can't be parsed.

while True:
    try:
        # Note: Python 2.x users should use raw_input, the equivalent of 3.x's input
        age = int(input("Please enter your age: "))
    except ValueError:
        print("Sorry, I didn't understand that.")
        #better try again... Return to the start of the loop
        continue
    else:
        #age was successfully parsed!
        #we're ready to exit the loop.
        break
if age >= 18: 
    print("You are able to vote in the United States!")
else:
    print("You are not able to vote in the United States.")

Implementing Your Own Validation Rules

If you want to reject values that Python can successfully parse, you can add your own validation logic.

while True:
    data = input("Please enter a loud message (must be all caps): ")
    if not data.isupper():
        print("Sorry, your response was not loud enough.")
        continue
    else:
        #we're happy with the value given.
        #we're ready to exit the loop.
        break

while True:
    data = input("Pick an answer from A to D:")
    if data.lower() not in ('a', 'b', 'c', 'd'):
        print("Not an appropriate choice.")
    else:
        break

Combining Exception Handling and Custom Validation

Both of the above techniques can be combined into one loop.

while True:
    try:
        age = int(input("Please enter your age: "))
    except ValueError:
        print("Sorry, I didn't understand that.")
        continue

    if age < 0:
        print("Sorry, your response must not be negative.")
        continue
    else:
        #age was successfully parsed, and we're happy with its value.
        #we're ready to exit the loop.
        break
if age >= 18: 
    print("You are able to vote in the United States!")
else:
    print("You are not able to vote in the United States.")

Encapsulating it All in a Function

If you need to ask your user for a lot of different values, it might be useful to put this code in a function, so you don't have to retype it every time.

def get_non_negative_int(prompt):
    while True:
        try:
            value = int(input(prompt))
        except ValueError:
            print("Sorry, I didn't understand that.")
            continue

        if value < 0:
            print("Sorry, your response must not be negative.")
            continue
        else:
            break
    return value

age = get_non_negative_int("Please enter your age: ")
kids = get_non_negative_int("Please enter the number of children you have: ")
salary = get_non_negative_int("Please enter your yearly earnings, in dollars: ")

Putting It All Together

You can extend this idea to make a very generic input function:

def sanitised_input(prompt, type_=None, min_=None, max_=None, range_=None):
    if min_ is not None and max_ is not None and max_ < min_:
        raise ValueError("min_ must be less than or equal to max_.")
    while True:
        ui = input(prompt)
        if type_ is not None:
            try:
                ui = type_(ui)
            except ValueError:
                print("Input type must be {0}.".format(type_.__name__))
                continue
        if max_ is not None and ui > max_:
            print("Input must be less than or equal to {0}.".format(max_))
        elif min_ is not None and ui < min_:
            print("Input must be greater than or equal to {0}.".format(min_))
        elif range_ is not None and ui not in range_:
            if isinstance(range_, range):
                template = "Input must be between {0.start} and {0.stop}."
                print(template.format(range_))
            else:
                template = "Input must be {0}."
                if len(range_) == 1:
                    print(template.format(*range_))
                else:
                    expected = " or ".join((
                        ", ".join(str(x) for x in range_[:-1]),
                        str(range_[-1])
                    ))
                    print(template.format(expected))
        else:
            return ui

With usage such as:

age = sanitised_input("Enter your age: ", int, 1, 101)
answer = sanitised_input("Enter your answer: ", str.lower, range_=('a', 'b', 'c', 'd'))

Common Pitfalls, and Why you Should Avoid Them

The Redundant Use of Redundant input Statements

This method works but is generally considered poor style:

data = input("Please enter a loud message (must be all caps): ")
while not data.isupper():
    print("Sorry, your response was not loud enough.")
    data = input("Please enter a loud message (must be all caps): ")

It might look attractive initially because it's shorter than the while True method, but it violates the Don't Repeat Yourself principle of software development. This increases the likelihood of bugs in your system. What if you want to backport to 2.7 by changing input to raw_input, but accidentally change only the first input above? It's a SyntaxError just waiting to happen.

Recursion Will Blow Your Stack

If you've just learned about recursion, you might be tempted to use it in get_non_negative_int so you can dispose of the while loop.

def get_non_negative_int(prompt):
    try:
        value = int(input(prompt))
    except ValueError:
        print("Sorry, I didn't understand that.")
        return get_non_negative_int(prompt)

    if value < 0:
        print("Sorry, your response must not be negative.")
        return get_non_negative_int(prompt)
    else:
        return value

This appears to work fine most of the time, but if the user enters invalid data enough times, the script will terminate with a RuntimeError: maximum recursion depth exceeded. You may think "no fool would make 1000 mistakes in a row", but you're underestimating the ingenuity of fools!

Self-reference for cell, column and row in worksheet functions

Just for row, but try referencing a cell just below the selected cell and subtracting one from row.

=ROW(A2)-1

Yields the Row of cell A1 (This formula would go in cell A1.

This avoids all the indirect() and index() use but still works.

Reflection: How to Invoke Method with parameters

I tried to work with all the suggested answers above but nothing seems to work for me. So i am trying to explain what worked for me here.

I believe if you are calling some method like the Main below or even with a single parameter as in your question, you just have to change the type of parameter from string to object for this to work. I have a class like below

//Assembly.dll
namespace TestAssembly{
    public class Main{

        public void Hello()
        { 
            var name = Console.ReadLine();
            Console.WriteLine("Hello() called");
            Console.WriteLine("Hello" + name + " at " + DateTime.Now);
        }

        public void Run(string parameters)
        { 
            Console.WriteLine("Run() called");
            Console.Write("You typed:"  + parameters);
        }

        public string TestNoParameters()
        {
            Console.WriteLine("TestNoParameters() called");
            return ("TestNoParameters() called");
        }

        public void Execute(object[] parameters)
        { 
            Console.WriteLine("Execute() called");
           Console.WriteLine("Number of parameters received: "  + parameters.Length);

           for(int i=0;i<parameters.Length;i++){
               Console.WriteLine(parameters[i]);
           }
        }

    }
}

Then you have to pass the parameterArray inside an object array like below while invoking it. The following method is what you need to work

private void ExecuteWithReflection(string methodName,object parameterObject = null)
{
    Assembly assembly = Assembly.LoadFile("Assembly.dll");
    Type typeInstance = assembly.GetType("TestAssembly.Main");

    if (typeInstance != null)
    {
        MethodInfo methodInfo = typeInstance.GetMethod(methodName);
        ParameterInfo[] parameterInfo = methodInfo.GetParameters();
        object classInstance = Activator.CreateInstance(typeInstance, null);

        if (parameterInfo.Length == 0)
        {
            // there is no parameter we can call with 'null'
            var result = methodInfo.Invoke(classInstance, null);
        }
        else
        {
            var result = methodInfo.Invoke(classInstance,new object[] { parameterObject } );
        }
    }
}

This method makes it easy to invoke the method, it can be called as following

ExecuteWithReflection("Hello");
ExecuteWithReflection("Run","Vinod");
ExecuteWithReflection("TestNoParameters");
ExecuteWithReflection("Execute",new object[]{"Vinod","Srivastav"});

Add custom buttons on Slick Carousel

A variation on the answer by @tallgirltaadaa , draw your own button in the shape of a caret:

var a = $('.MyCarouselContainer').slick({
    prevArrow: '<canvas class="prevArrowCanvas a-left control-c prev slick-prev" width="15" height="50"></canvas>',
    nextArrow: '<canvas class="nextArrowCanvas a-right control-c next slick-next" width="15" height="50"></canvas>'
});

function drawNextPreviousArrows(strokeColor) {
    var c = $(".prevArrowCanvas")[0];
    var ctx = c.getContext("2d");
    ctx.clearRect(0, 0, c.width, c.height);
    ctx.moveTo(15, 0);
    ctx.lineTo(0, 25);
    ctx.lineTo(15, 50);
    ctx.lineWidth = 2;
    ctx.strokeStyle = strokeColor;
    ctx.stroke();
    var c = $(".nextArrowCanvas")[0];
    var ctx = c.getContext("2d");
    ctx.clearRect(0, 0, c.width, c.height);
    ctx.moveTo(0, 0);
    ctx.lineTo(15, 25);
    ctx.lineTo(0, 50);
    ctx.lineWidth = 2;
    ctx.strokeStyle = strokeColor;
    ctx.stroke();
}
drawNextPreviousArrows("#cccccc");

then add the css

.slick-prev, .slick-next 
    height: 50px;s
}

How to launch another aspx web page upon button click?

If you'd like to use Code Behind, may I suggest the following solution for an asp:button -

ASPX Page

<asp:Button ID="btnRecover" runat="server" Text="Recover" OnClick="btnRecover_Click" />

Code Behind

    protected void btnRecover_Click(object sender, EventArgs e)
    {
        var recoveryId = Guid.Parse(lbRecovery.SelectedValue);
        var url = string.Format("{0}?RecoveryId={1}", @"../Recovery.aspx", vehicleId);

        // Response.Redirect(url); // Old way

        Response.Write("<script> window.open( '" + url + "','_blank' ); </script>");
        Response.End();
    }

Fatal error: Can't open and lock privilege tables: Table 'mysql.host' doesn't exist

When download mysql zip version, if run mysqld directly, you'll get this error: 2016-02-18T07:23:48.318481Z 0 [ERROR] Fatal error: Can't open and lock privilege tables: Table 'mysql.user' doesn't exist 2016-02-18T07:23:48.319482Z 0 [ERROR] Aborting

You have to run below command first: mysqld --initialize

Make sure your data folder is empty before this command.

Sass and combined child selector

Without the combined child selector you would probably do something similar to this:

foo {
  bar {
    baz {
      color: red;
    }
  }
}

If you want to reproduce the same syntax with >, you could to this:

foo {
  > bar {
    > baz {
      color: red;
    }
  }
}

This compiles to this:

foo > bar > baz {
  color: red;
}

Or in sass:

foo
  > bar
    > baz
      color: red

How do I install a module globally using npm?

On a Mac, I found the output contained the information I was looking for:

$> npm install -g karma
...
...
> [email protected] install /usr/local/share/npm/lib/node_modules/karma/node_modules/socket.io/node_modules/socket.io-client/node_modules/ws
> (node-gyp rebuild 2> builderror.log) || (exit 0)
...
$> ls /usr/local/share/npm/bin
karma nf

After adding /usr/local/share/npm/bin to the export PATH line in my .bash_profile, saving it, and sourceing it, I was able to run

$> karma --help

normally.

UnsupportedClassVersionError: JVMCFRE003 bad major version in WebSphere AS 7

In this Eclipse Preferences panel you can change the compiler compatibility from 1.7 to 1.6. This solved the similar message I was getting. For Eclipse, it is under: Preferences -> Java -> Compiler: 'Compiler compliance level'

How do I set proxy for chrome in python webdriver?

from selenium import webdriver

PROXY = "23.23.23.23:3128" # IP:PORT or HOST:PORT

chrome_options = webdriver.ChromeOptions()
chrome_options.add_argument('--proxy-server=%s' % PROXY)

chrome = webdriver.Chrome(options=chrome_options)
chrome.get("http://whatismyipaddress.com")

Is there an equivalent to background-size: cover and contain for image elements?

Assuming you can arrange to have a container element you wish to fill, this appears to work, but feels a bit hackish. In essence, I just use min/max-width/height on a larger area and then scale that area back into the original dimensions.

_x000D_
_x000D_
.container {_x000D_
  width: 800px;_x000D_
  height: 300px;_x000D_
  border: 1px solid black;_x000D_
  overflow:hidden;_x000D_
  position:relative;_x000D_
}_x000D_
.container.contain img {_x000D_
  position: absolute;_x000D_
  left:-10000%; right: -10000%; _x000D_
  top: -10000%; bottom: -10000%;_x000D_
  margin: auto auto;_x000D_
  max-width: 10%;_x000D_
  max-height: 10%;_x000D_
  -webkit-transform:scale(10);_x000D_
  transform: scale(10);_x000D_
}_x000D_
.container.cover img {_x000D_
  position: absolute;_x000D_
  left:-10000%; right: -10000%; _x000D_
  top: -10000%; bottom: -10000%;_x000D_
  margin: auto auto;_x000D_
  min-width: 1000%;_x000D_
  min-height: 1000%;_x000D_
  -webkit-transform:scale(0.1);_x000D_
  transform: scale(0.1);_x000D_
}
_x000D_
<h1>contain</h1>_x000D_
  <div class="container contain">_x000D_
    <img _x000D_
       src="https://www.google.de/logos/doodles/2014/european-parliament-election-2014-day-4-5483168891142144-hp.jpg" _x000D_
       />_x000D_
    <!-- 366x200 -->_x000D_
  </div>_x000D_
  <h1>cover</h1>_x000D_
  <div class="container cover">_x000D_
    <img _x000D_
       src="https://www.google.de/logos/doodles/2014/european-parliament-election-2014-day-4-5483168891142144-hp.jpg" _x000D_
       />_x000D_
    <!-- 366x200 -->_x000D_
  </div>
_x000D_
_x000D_
_x000D_

How to take off line numbers in Vi?

From the Document "Mastering the VI editor":

number (nu)
Displays lines with line numbers on the left side.

Disabling of EditText in Android

In my case I needed my EditText to scroll text if no. of lines exceed maxLines when its disabled. This implementation worked perfectly for me.

private void setIsChatEditTextEditable(boolean value)
{
    if(value)
    {
        mEdittext.setCursorVisible(true);
        mEdittext.setSelection(chat_edittext.length());
       // use new EditText(getApplicationContext()).getKeyListener()) if required below
        mEdittext.setKeyListener(new AppCompatEditText(getApplicationContext()).getKeyListener());  
    }
    else
    {
        mEdittext.setCursorVisible(false);
        mEdittext.setKeyListener(null);
    }
}

C# code to validate email address

.net 4.5 added System.ComponentModel.DataAnnotations.EmailAddressAttribute

You can browse the EmailAddressAttribute's source, this is the Regex it uses internally:

const string pattern = @"^((([a-z]|\d|[!#\$%&'\*\+\-\/=\?\^_`{\|}~]|[\u00A0-\uD7FF\uF900-\uFDCF\uFDF0-\uFFEF])+(\.([a-z]|\d|[!#\$%&'\*\+\-\/=\?\^_`{\|}~]|[\u00A0-\uD7FF\uF900-\uFDCF\uFDF0-\uFFEF])+)*)|((\x22)((((\x20|\x09)*(\x0d\x0a))?(\x20|\x09)+)?(([\x01-\x08\x0b\x0c\x0e-\x1f\x7f]|\x21|[\x23-\x5b]|[\x5d-\x7e]|[\u00A0-\uD7FF\uF900-\uFDCF\uFDF0-\uFFEF])|(\\([\x01-\x09\x0b\x0c\x0d-\x7f]|[\u00A0-\uD7FF\uF900-\uFDCF\uFDF0-\uFFEF]))))*(((\x20|\x09)*(\x0d\x0a))?(\x20|\x09)+)?(\x22)))@((([a-z]|\d|[\u00A0-\uD7FF\uF900-\uFDCF\uFDF0-\uFFEF])|(([a-z]|\d|[\u00A0-\uD7FF\uF900-\uFDCF\uFDF0-\uFFEF])([a-z]|\d|-|\.|_|~|[\u00A0-\uD7FF\uF900-\uFDCF\uFDF0-\uFFEF])*([a-z]|\d|[\u00A0-\uD7FF\uF900-\uFDCF\uFDF0-\uFFEF])))\.)+(([a-z]|[\u00A0-\uD7FF\uF900-\uFDCF\uFDF0-\uFFEF])|(([a-z]|[\u00A0-\uD7FF\uF900-\uFDCF\uFDF0-\uFFEF])([a-z]|\d|-|\.|_|~|[\u00A0-\uD7FF\uF900-\uFDCF\uFDF0-\uFFEF])*([a-z]|[\u00A0-\uD7FF\uF900-\uFDCF\uFDF0-\uFFEF])))\.?$";

error TS2339: Property 'x' does not exist on type 'Y'

The correct fix is to add the property in the type definition as explained in @Nitzan Tomer's answer. If that's not an option though:

(Hacky) Workaround 1

You can assign the object to a constant of type any, then call the 'non-existing' property.

const newObj: any = oldObj;
return newObj.someProperty;

You can also cast it as any:

return (oldObj as any).someProperty;

This fails to provide any type safety though, which is the point of TypeScript.


(Hacky) Workaround 2

Another thing you may consider, if you're unable to modify the original type, is extending the type like so:

interface NewType extends OldType {
  someProperty: string;
}

Now you can cast your variable as this NewType instead of any. Still not ideal but less permissive than any, giving you more type safety.

return (oldObj as NewType).someProperty;

expected assignment or function call: no-unused-expressions ReactJS

In my case it is happened due to curly braces of function if you use jsx then you need to change curly braces to Parentheses, see below code

const [countries] = useState(["USA", "UK", "BD"])

I tried this but not work, don't know why

 {countries.map((country) => {
        <MenuItem value={country}>{country}</MenuItem>
  })}

But when I change Curly Braces to parentheses and Its working fine for me

  {countries.map((country) => ( //Changes is here instead of {
        <MenuItem value={country}>{country}</MenuItem>
  ))} //and here instead of }
             

Hopefully it will help you too...

Docker compose port mapping

It seems like the other answers here all misunderstood your question. If I understand correctly, you want to make requests to localhost:6379 (the default for redis) and have them be forwarded, automatically, to the same port on your redis container.

https://unix.stackexchange.com/a/101906/38639 helped me get to the right answer.

First, you'll need to install the nc command on your image. On CentOS, this package is called nmap-ncat, so in the example below, just replace this with the appropriate package if you are using a different OS as your base image.

Next, you'll need to tell it to run a certain command each time the container boots up. You can do this using CMD.

# Add this to your Dockerfile
RUN yum install -y --setopt=skip_missing_names_on_install=False nmap-ncat
COPY cmd.sh /usr/local/bin/cmd.sh
RUN chmod +x /usr/local/bin/cmd.sh
CMD ["/usr/local/bin/cmd.sh"]

Finally, we'll need to set up port-forwarding in cmd.sh. I found that nc, even with the -l and -k options, will occasionally terminate when a request is completed, so I'm using a while-loop to ensure that it's always running.

# cmd.sh
#! /usr/bin/env bash

while nc -l -p 6379 -k -c "nc redis 6379" || true; do true; done &

tail -f /dev/null # Or any other command that never exits

How to logout and redirect to login page using Laravel 5.4?

Best way for Laravel 5.8

100% worked

Add this function inside your Auth\LoginController.php

use Illuminate\Http\Request;

And also add this

public function logout(Request $request)
{
    $this->guard()->logout();

    $request->session()->invalidate();

    return $this->loggedOut($request) ?: redirect('/login');
}

Enable IIS7 gzip

Global Gzip in HttpModule

If you don't have access to the final IIS instance (shared hosting...) you can create a HttpModule that adds this code to every HttpApplication.Begin_Request event :

HttpContext context = HttpContext.Current;
context.Response.Filter = new GZipStream(context.Response.Filter, CompressionMode.Compress);
HttpContext.Current.Response.AppendHeader("Content-encoding", "gzip");
HttpContext.Current.Response.Cache.VaryByHeaders["Accept-encoding"] = true;

Testing

Kudos, no solution is done without testing. I like to use the Firefox plugin "Liveheaders" it shows all the information about every http message between the browser and server, including compression, file size (which you could compare to the file size on the server).

NullInjectorError: No provider for AngularFirestore

I solved this problem by just removing firestore from:

import { AngularFirestore } from '@angular/fire/firestore/firestore';

in my component.ts file. as use only:

import { AngularFirestore } from '@angular/fire/firestore';

this can be also your problem.

Angles between two n-dimensional vectors in Python

Using numpy and taking care of BandGap's rounding errors:

from numpy.linalg import norm
from numpy import dot
import math

def angle_between(a,b):
  arccosInput = dot(a,b)/norm(a)/norm(b)
  arccosInput = 1.0 if arccosInput > 1.0 else arccosInput
  arccosInput = -1.0 if arccosInput < -1.0 else arccosInput
  return math.acos(arccosInput)

Note, this function will throw an exception if one of the vectors has zero magnitude (divide by 0).

data type not understood

Try:

mmatrix = np.zeros((nrows, ncols))

Since the shape parameter has to be an int or sequence of ints

http://docs.scipy.org/doc/numpy/reference/generated/numpy.zeros.html

Otherwise you are passing ncols to np.zeros as the dtype.

Convert timedelta to total seconds

Use timedelta.total_seconds().

>>> import datetime
>>> datetime.timedelta(seconds=24*60*60).total_seconds()
86400.0

C++: Converting Hexadecimal to Decimal

Here is a solution using strings and converting it to decimal with ASCII tables:

#include <iostream>
#include <string>
#include "math.h"
using namespace std;
unsigned long hex2dec(string hex)
{
    unsigned long result = 0;
    for (int i=0; i<hex.length(); i++) {
        if (hex[i]>=48 && hex[i]<=57)
        {
            result += (hex[i]-48)*pow(16,hex.length()-i-1);
        } else if (hex[i]>=65 && hex[i]<=70) {
            result += (hex[i]-55)*pow(16,hex.length( )-i-1);
        } else if (hex[i]>=97 && hex[i]<=102) {
            result += (hex[i]-87)*pow(16,hex.length()-i-1);
        }
    }
    return result;
}

int main(int argc, const char * argv[]) {
    string hex_str;
    cin >> hex_str;
    cout << hex2dec(hex_str) << endl;
    return 0;
}

Can an Android Toast be longer than Toast.LENGTH_LONG?

Toast.makeText(this, "Text", Toast.LENGTH_LONG).show(); 
Toast.makeText(this, "Text", Toast.LENGTH_LONG).show();

A very simple solution to the question. Twice or triple of them will make Toast last longer. It is the only way around.

How to pass complex object to ASP.NET WebApi GET from jQuery ajax call?

After finding this StackOverflow question/answer

Complex type is getting null in a ApiController parameter

the [FromBody] attribute on the controller method needs to be [FromUri] since a GET does not have a body. After this change the "filter" complex object is passed correctly.

Replace one character with another in Bash

Try this for paths:

echo \"hello world\"|sed 's/ /+/g'|sed 's/+/\/g'|sed 's/\"//g'

It replaces the space inside the double-quoted string with a + sing, then replaces the + sign with a backslash, then removes/replaces the double-quotes.

I had to use this to replace the spaces in one of my paths in Cygwin.

echo \"$(cygpath -u $JAVA_HOME)\"|sed 's/ /+/g'|sed 's/+/\\/g'|sed 's/\"//g'

Change div height on button click

Look at to vwphillips' post from 03-06-2010, 03:35 PM in http://www.codingforums.com/archive/index.php/t-190887.html

<html>
   <head>
      <title></title>
      <meta http-equiv="Content-Type" content="text/html; charset=iso-8859-1">

      <script type="text/javascript">

        function Div(id,ud) {
           var div=document.getElementById(id);
           var h=parseInt(div.style.height)+ud;
           if (h>=1){
              div.style.height = h + "em"; // I'm using "em" instead of "px", but you can use px like measure...
           }
        }
      </script>

   </head>
   <body>
      <div>
         <input type="button" value="+" onclick="Div('my_div', 1);">&nbsp;&nbsp; 
         <input type="button" value="-" onclick="Div('my_div', -1);"></div>
      </div>

      <div id="my_div" style="height: 1em; width: 1em; overflow: auto;"></div>

   </body>
</html>

This worked for me :)

Best regards!

Difference between Convert.ToString() and .ToString()

To understand both the methods let's take an example:

int i =0;
MessageBox.Show(i.ToString());
MessageBox.Show(Convert.ToString(i)); 

Here both the methods are used to convert the string but the basic difference between them is: Convert function handles NULL, while i.ToString() does not it will throw a NULL reference exception error. So as good coding practice using convert is always safe.

Let's see another example:

string s;
object o = null;
s = o.ToString(); 
//returns a null reference exception for s. 

string s;
object o = null;
s = Convert.ToString(o); 
//returns an empty string for s and does not throw an exception.

How can I check if a directory exists?

You might use stat() and pass it the address of a struct stat, then check its member st_mode for having S_IFDIR set.

#include <stdio.h>
#include <sys/types.h>
#include <sys/stat.h>
#include <unistd.h>

...

char d[] = "mydir";

struct stat s = {0};

if (!stat(d, &s))
  printf("'%s' is %sa directory.\n", d, (s.st_mode & S_IFDIR)  : "" ? "not ");
  // (s.st_mode & S_IFDIR) can be replaced with S_ISDIR(s.st_mode)
else
  perror("stat()");

Deciding between HttpClient and WebClient

I have benchmark between HttpClient, WebClient, HttpWebResponse then call Rest Web Api

and result Call Rest Web Api Benchmark

---------------------Stage 1  ---- 10 Request

{00:00:17.2232544} ====>HttpClinet
{00:00:04.3108986} ====>WebRequest
{00:00:04.5436889} ====>WebClient

---------------------Stage 1  ---- 10 Request--Small Size
{00:00:17.2232544}====>HttpClinet
{00:00:04.3108986}====>WebRequest
{00:00:04.5436889}====>WebClient

---------------------Stage 3  ---- 10 sync Request--Small Size
{00:00:15.3047502}====>HttpClinet
{00:00:03.5505249}====>WebRequest
{00:00:04.0761359}====>WebClient

---------------------Stage 4  ---- 100 sync Request--Small Size
{00:03:23.6268086}====>HttpClinet
{00:00:47.1406632}====>WebRequest
{00:01:01.2319499}====>WebClient

---------------------Stage 5  ---- 10 sync Request--Max Size

{00:00:58.1804677}====>HttpClinet    
{00:00:58.0710444}====>WebRequest    
{00:00:38.4170938}====>WebClient
    
---------------------Stage 6  ---- 10 sync Request--Max Size

{00:01:04.9964278}====>HttpClinet    
{00:00:59.1429764}====>WebRequest    
{00:00:32.0584836}====>WebClient

_____ WebClient Is faster ()

var stopWatch = new Stopwatch();
        stopWatch.Start();
        for (var i = 0; i < 10; ++i)
        {
            CallGetHttpClient();
            CallPostHttpClient();
        }

        stopWatch.Stop();

        var httpClientValue = stopWatch.Elapsed;

        stopWatch = new Stopwatch();

        stopWatch.Start();
        for (var i = 0; i < 10; ++i)
        {
            CallGetWebRequest();
            CallPostWebRequest();
        }

        stopWatch.Stop();

        var webRequesttValue = stopWatch.Elapsed;


        stopWatch = new Stopwatch();

        stopWatch.Start();
        for (var i = 0; i < 10; ++i)
        {

            CallGetWebClient();
            CallPostWebClient();

        }

        stopWatch.Stop();

        var webClientValue = stopWatch.Elapsed;

//-------------------------Functions

private void CallPostHttpClient()
    {
        var httpClient = new HttpClient();
        httpClient.BaseAddress = new Uri("https://localhost:44354/api/test/");
        var responseTask = httpClient.PostAsync("PostJson", null);
        responseTask.Wait();

        var result = responseTask.Result;
        var readTask = result.Content.ReadAsStringAsync().Result;

    }
    private void CallGetHttpClient()
    {
        var httpClient = new HttpClient();
        httpClient.BaseAddress = new Uri("https://localhost:44354/api/test/");
        var responseTask = httpClient.GetAsync("getjson");
        responseTask.Wait();

        var result = responseTask.Result;
        var readTask = result.Content.ReadAsStringAsync().Result;

    }
    private string CallGetWebRequest()
    {
        var request = (HttpWebRequest)WebRequest.Create("https://localhost:44354/api/test/getjson");

        request.Method = "GET";
        request.AutomaticDecompression = DecompressionMethods.Deflate | DecompressionMethods.GZip;

        var content = string.Empty;

        using (var response = (HttpWebResponse)request.GetResponse())
        {
            using (var stream = response.GetResponseStream())
            {
                using (var sr = new StreamReader(stream))
                {
                    content = sr.ReadToEnd();
                }
            }
        }

        return content;
    }
    private string CallPostWebRequest()
    {

        var apiUrl = "https://localhost:44354/api/test/PostJson";


        HttpWebRequest httpRequest = (HttpWebRequest)WebRequest.Create(new Uri(apiUrl));
        httpRequest.ContentType = "application/json";
        httpRequest.Method = "POST";
        httpRequest.ContentLength = 0;

        using (var httpResponse = (HttpWebResponse)httpRequest.GetResponse())
        {
            using (Stream stream = httpResponse.GetResponseStream())
            {
                var json = new StreamReader(stream).ReadToEnd();
                return json;
            }
        }

        return "";
    }

    private string CallGetWebClient()
    {
        string apiUrl = "https://localhost:44354/api/test/getjson";


        var client = new WebClient();

        client.Headers["Content-type"] = "application/json";

        client.Encoding = Encoding.UTF8;

        var json = client.DownloadString(apiUrl);


        return json;
    }

    private string CallPostWebClient()
    {
        string apiUrl = "https://localhost:44354/api/test/PostJson";


        var client = new WebClient();

        client.Headers["Content-type"] = "application/json";

        client.Encoding = Encoding.UTF8;

        var json = client.UploadString(apiUrl, "");


        return json;
    }

What does the SQL Server Error "String Data, Right Truncation" mean and how do I fix it?

Either the parameter supplied for ZIP_CODE is larger (in length) than ZIP_CODEs column width or the parameter supplied for CITY is larger (in length) than CITYs column width.

It would be interesting to know the values supplied for the two ? placeholders.

React.js: Set innerHTML vs dangerouslySetInnerHTML

Based on (dangerouslySetInnerHTML).

It's a prop that does exactly what you want. However they name it to convey that it should be use with caution

Java: How to Indent XML Generated by Transformer

The following code is working for me with Java 7. I set the indent (yes) and indent-amount (2) on the transformer (not the transformer factory) to get it working.

TransformerFactory tf = TransformerFactory.newInstance();
Transformer t = tf.newTransformer();
t.setOutputProperty("{http://xml.apache.org/xslt}indent-amount", "2");
t.setOutputProperty(OutputKeys.INDENT, "yes");
t.transform(source, result);

@mabac's solution to set the attribute didn't work for me, but @lapo's comment proved helpful.

CSS transition shorthand with multiple properties?

If you have several specific properties that you want to transition in the same way (because you also have some properties you specifically don't want to transition, say opacity), another option is to do something like this (prefixes omitted for brevity):

.myclass {
    transition: all 200ms ease;
    transition-property: box-shadow, height, width, background, font-size;
}

The second declaration overrides the all in the shorthand declaration above it and makes for (occasionally) more concise code.

_x000D_
_x000D_
/* prefixes omitted for brevity */_x000D_
.box {_x000D_
    height: 100px;_x000D_
    width: 100px;_x000D_
    background: red;_x000D_
    box-shadow: red 0 0 5px 1px;_x000D_
    transition: all 500ms ease;_x000D_
    /*note: not transitioning width */_x000D_
    transition-property: height, background, box-shadow;_x000D_
}_x000D_
_x000D_
.box:hover {_x000D_
  height: 50px;_x000D_
  width: 50px;_x000D_
  box-shadow: blue 0 0 10px 3px;_x000D_
  background: blue;_x000D_
}
_x000D_
<p>Hover box for demo</p>_x000D_
<div class="box"></div>
_x000D_
_x000D_
_x000D_

Demo

onclick event function in JavaScript

I suggest you do: <input type="button" value="button text" onclick="click()"> Hope this helps you!

What determines the monitor my app runs on?

Important note: If you remember the position of your application and shutdown and then start up again at that position, keep in mind that the user's monitor configuration may have changed while your application was closed.

Laptop users, for example, frequently change their display configuration. When docked there may be a 2nd monitor that disappears when undocked. If the user closes an application that was running on the 2nd monitor and the re-opens the application when the monitor is disconnected, restoring the window to the previous coordinates will leave it completely off-screen.

To figure out how big the display really is, check out GetSystemMetrics.

Automatically resize jQuery UI dialog to the width of the content loaded by ajax

I imagine setting float:left for the dialog will work. Presumably the dialog is absolutely positioned by the plugin, in which case its position will be determined by this, but the side-effect of float - making elements only as wide as they need to be to hold content - will still take effect.

This should work if you just put a rule like

.myDialog {float:left}

in your stylesheet, though you may need to set it using jQuery

Insert data into hive table

If you already have a table pre_loaded_tbl with some data. You can use a trick to load the data into your table with following query

INSERT INTO TABLE tweet_table 
  SELECT  "my_data" AS my_column 
    FROM   pre_loaded_tbl 
   LIMIT   5;

Also please note that "my_data" is independent of any data in the pre_loaded_tbl. You can select any data and write any column name (here my_data and my_column). Hive does not require it to have same column name. However structure of select statement should be same as that of your tweet_table. You can use limit to determine how many times you can insert into the tweet_table.

However if you haven't' created any table, you will have to load the data using file copy or load data commands in above answers.